AIIMS PG May 2018


Kindly Choose the Subject from the Above Menu Bar to show the questions of that particular subject from AIIMS PG May 2018.

Anatomy

Question.1
Type I collagen is present in all except

AIIMS PG May-2018
A. Bone
B. Cartilage
C. Ligament
D. Aponeurosis
Correct Ans: B
Explanation

Ans. b. Cartilage

  • Ref: Ross & Pawlina, 7h ed., Histologt A Text and Atlas, pg. 160-167 and Junqueira’s Basic Histologt, 13’h ed-, pg. 106-108
Type Location Functions
1

Connective tissue of skin,

bone, tendon, ligaments,

dentin, sclera, fascia, and

organ capsules (accounts for

Provides resistance

to force, tension, and

stretch


Question.2
Which of the following is the marked structure in the below picture?

AIIMS PG May-2018
A. Neurotransmitter
B. Synaptic vesicles
C. Microtubule
D. Collagen fibril
Correct Ans: B
Explanation

Ans. b. Synaptic vesicles

Ref: Z. Taoufiq, OIST 2013

  • An ultramicroscopic image shows an isolated synapse from a brain sample before mass spectrometry analyses.
  • The pre-synapse typically shows many vesicles containing neurotransmitters kept attached to the post-synapse.

Question.3
Compartment of  leg without neurovascular bundle?

AIIMS PG May-2018
A. Anterior
B. Lateral
C. Deep posterior
D. Superficial posterior
Correct Ans: D
Explanation

Ans. d. Superficial posterior

Ref: Grays Anilomy, 4l” ed, pg, 1406-1412 and Last anatomy I2’h ed-, pg. 142

Contents of flexor (posterior) compartment:

Superficial part:

  • Gastrocnemius, plantaris and soleus muscles

Deep part:

  1. Popliteus, Flexor digitorum longus, flexor hallucis longus and Tibialis pos-terior muscle
  2. Posterior tibial and peroneal vessels
  3. Tibial nerve

Question.4
Spinal cord ends at what level in adults?

AIIMS PG May-2018
A. T12
B. L1
C. L2
D. L3
Correct Ans: B
Explanation

Ans. b. L1 
Ref: Grays Anatomy, 4l” ed., pg. 762-763

  • In the adult, the spinal cord terminates on average at the level of the middle third of the body of the first lumbar vertebra which corresponds approximately to the transpyloric plane.
  • Spinal cord extends from C I to L I (lower border) in adults.
  • Filum terminale extends from lower end of spinal cord to the tip of coccyx.
  • Dural sheath extends up to 52 vertebrae.
  • Subdural space extends up to 52.
  • Subarachnoid space extends up to 52.
  • Piamater extends up to tip of coccyx.

Question.5
Inferiorthyroid artery is a branch ol?

AIIMS PG May-2018
A. Thyrocervical trunk
B. ICA
C. Costocervical trunk
D. ECA
Correct Ans: A
Explanation

Ans. A. Thyrocervical trunk
Ref: Grays Anatomy,4Ia ed., pg. 455458

  • Inferior thyroid artery is branch of thyrocervical trunk of subclavian artery.
  • The thyrocervical trunk arises fiom the front of the first part of the subclavian artery near the medial border of scalenus anterior, and divides into the inferior thyroid, suprascapular and superficial cervical arteries.

Question.6
Which of the following sensation is not perceived by spinal nucleus of trigeminal nerve?

AIIMS PG May-2018
A. Pain
B. Touch
C. Temperature
D. Proprioception
Correct Ans: D
Explanation

Ans. D. Proprioception
[Ref: Barr’s the haman nervons system, I0’h ed., pg. 127 & Snell’s neurounatomy 7’t’ed., pg. 342-343]

  • The trigeminal nerve is the largest cranial nerve and contains both sensory and motor fibres.
  • Sensory nerve to greater part of the head.
  • Motor nerve to several muscles, including the muscles of mastication.
  • The trigeminal nerve has four nuclei:
  • (1) the main sensory nucleus, (2) the spinal nucleus, (3) the mesencephalic nucleus, and (4) the motor nucleus.
  • Proprioceptive impulses from the muscles of mastication and from the facial and extraocular muscles are carried by fibres in the sensory root of the trigeminal nerve that have bypassed the semilunar or trigeminal ganglion.
  • The fibres cells of origin are the unipolar cells of the mesencephalic nucleus.

Question.7
Purkinje fibers relay to which of the following?

AIIMS PG May-2018
A. Dentate nucleus
B. Caudate nucleus
C. Amygdala
D. VLN thalamus
Correct Ans: A
Explanation

Ans. A. Dentate nucleus
[Ref: Barr’s human nervous system 10th ed., pg. 164 and Snell’s neuroanatomy, 7 e4, W.235]

  • The cerebellum is composed of an outer covering of gray matter called the cortex and inner white matter.
  • There is a large amount of white matter in each cerebellar hernisphere.
  • The efferent fibres constitute the output of the cerebellum and commence as the axons of the Purkinje cells of the cerebellar cortex.
  • The great majorify of the Purkinje cell axons pass to and synapse with the neurons of the cerebellar nuclei (f’astigial, globose, embolifbrm, and dentate).

Question.8
Which of the following is false regarding blood supply of dura mater?

AIIMS PG May-2018
A. Meningeal branch of ICA in posterior cranial tbssa
B. Accessory meningeal artery
C. Meningeal branch of anterior and posterior ethmoidal artery
D. Middle meningeal artery
Correct Ans: C
Explanation

Ans. C. Meningeal branch of anterior and posterior ethmoidal artery
[Ref Grays Anatomy, 41″ ed., pg. 439 und Snell’s neuroanatomy, Vt’ed., pg. 175-477]
Anterior cranial fossa:

  • Anterior rneningeal branches of the anterior and posterior ethmoidal and internal carotid arleries and a branch of the nriddle meningeal artery.

Middle cranial fossa:

  • Middle and accessory meningeal branches of the maxillary artery, a branch of the ascending pharyngeal artery (entering via the tbrarnen lacerum), branches of the internal carotid and a recurrent branch of the lacrimal artery.

Posterior fossa:

  • Meningeal branches of the occipital artery, posterior meningeal branches of vertebral artery, occasional small branches of the ascending pharyngeal artery, hypoglossal canal & dorsal meningeal and tentorial arteries arising from the meningohypophysial trunk.

Question.9
Which of the following is true about nerve supply of adrenal gland?

AIIMS PG May-2018
A. Release of catecholamines is not affected by nerve supply to adrenals
B. Preganglionic fibres from lower thoracic and lumbar vertebra come via sympathetic chain to supply adrenals
C. Adrenal cortex doesn’t have nerve supply as it has endocrine functions
D. Adrenal medulla doesn’t have nerve supply
Correct Ans: B
Explanation

Ans. B. Preganglionic fibres from lower thoracic and lumbar vertebra come via sympathetic chain to supply adrenals
[Ref: Grays Anatomy, 41″ ed., pS. 439]

  • Suprarenal gland – Greater autonomic supply than any other organ.
  • The nerves are distributed throughout the gland around blood vessels (regulating blood flow), in the medulla (stimulating the release of catecholamines from chromaffin cells), and in the cortex (where they may influence steroid hormone production).
  • A suprarenal plexus lies between the medial aspect ofeach gland and the coeliac and aorticorenal ganglia.
  • It contains predominantly preganglionic syrnpathetic fibres that originate in the lower thoracic spinal segments, reach the plexus via branches of the greater splanchnic newes, and synapse on clusters of large medullary chromaffin cells.

Question.10
Compartment of  leg without neurovascular bundle?

AIIMS PG May-2018
A. Anterior
B. Lateral
C. Deep posterior
D. Superficial posterior
Correct Ans: D
Explanation

Ans. d. Superficial posterior

[Ref: Grays Anilomy, 4l” ed, pg, 1406-1412 and Last anatomy I2’h ed-, pg. 142]

Contents of flexor (posterior) compartment:

Superficial part:

  • Gastrocnemius, plantaris and soleus muscles

Deep part:

  1. Popliteus, Flexor digitorum longus, flexor hallucis longus and Tibialis pos-terior muscle
  2. Posterior tibial and peroneal vessels
  3. Tibial nerve

Physiology

Question.1
GFR is increased by all except?

AIIMS PG May-2018
A. lncreased renal blood flow
B. Efferent arteriole constriction
C. Renal stone in ureter
D. Decreased oncotic pressure
Correct Ans: C
Explanation

Ans. C. Renal stone in ureter
[Ref: Ganong’s Review of Medical Physiologt, 2!n ed., ch -7, pg. 677-78]

  • GFR = Kf [(PGC – PT) – (nGC – ztT)]
  • K.: Glomerular Ultrafiltration coelicient
  • P: Hydrostatic pressure,
  • lI: Oncotic Pressure,
  • CC: GIomerular capillaries
  • T: tubule
  • lncreased Renal blood flow. efferent arteriole constriction : Increase GC hydrostatic pressure
  • Decrease oncotic pressure in plasma : increased GFR

Renal stones:

  • Tubular Obstruction can lead to increased hydrostatic pressure in tubule (bowman’s capsule).

Question.2
Baroreceptor is which type of feedback?

AIIMS PG May-2018
A. Negative feedback
B. Positive feedback
C. Both negative and positive
D. Feed forward control
Correct Ans: A
Explanation

Ans. A. Negative feedback
[Ref: Ganong’s Review of Medical Physiologlt, 25n ed., ch-32, pg. 587]

  • Baroreflex senses arterial pressure by means ofwall stress at the baroreceptors and acts in a feedback way to control bp.

Examples of various control systems:
Positive Feedback

  • Estrogen control of GnRH release at ovulation

Negative Feedback

  • Blood pressure
  • Plasma osmolarity.
  • Plasma CO2, pH.
  • Endocrine gland secretion

Feed forward Control:

  • Temperature regulation

Question.3
Cell membrane of a cell is freely permeable to three ions X, Y and Z. At resting membrane potential, the respective equilibrium potentials of Xand Y are -30mV and -40 mV.What will happen to the membrane potential if a drug is infused which can block the permeability of Z?

AIIMS PG May-2018
A. Depolarisation
B. Hyperpolarization
C. No change
D. Hyperpolarization following repolarization
Correct Ans: A
Explanation

Ans. A. Depolarisation
[Ref: Guyton & Hall 13′ ed., ch-5, pg. 61-63]

  • In this question if Z has a reversal potential or original RMP was more negative (e.g., -70 or -90 mV) then closure of C ion channels would shift the RMP closer to reversal potential of A, B thus depolarising the cell

Question.4
Which of the following is the diluting segment of kidney?

AIIMS PG May-2018
A. PCT
B. Collecting duct
C. Ascending thick loop Henle
D. Descending loop of Henle
Correct Ans: C
Explanation

Ans. C. Ascending thick loop Henle
[Ref: Ganong’s Review of Medical Physiology, 2lh ed., ch-37, pg. 684]

  • TAL is permeable to solutes like Na+ and Cl- and impermeable to water.
  • The tubular fluid is diluted and becomes hypotonic as it reaches the top of the TAL.


Question.5

How an electrical synapse is different from a chemical synapse?

AIIMS PG May-2018
A. No cytoplasmic continuity between presynaptic and post synaptic
B. Direction of impulse is unidirectional
C. Connexons are involved in electrical synapse
D. Electrical synapse has more latent period than chemical synapse
Correct Ans: C
Explanation

Ans. C. Connexons are involved in electrical synapse
[Ref: Ganong’s Review of Medical Physiologt, 25’h ed., ch-2, pg. 4l-42]

  • Electrical synapses are composed of gap junctions which establish intercellular cytoplasmic continuity between cells for the flow of ions and messengers.
  • Transmission across electrical synapses is faster as chemical synapses are associated with a synaptic delay created by neurotransmitter release, binding and activation of second messenger systems.
  • Electrical synapses are bidirectional in nature.
  • Gap junctions are made of connexons which compose 6 connexin subunits.
  • Two connexons aligned opposite to each other form a gap junction.

Biochemistry

Question.1
Vitamin C cannot be produced in humans due to lack of:

AIIMS PG May-2018
A. L-gulonolactone oxidase
B. Xylutitol reductase
C. Pyruvate dehydrogenase
D. UDP glucose dehydrogenase
Correct Ans: A
Explanation

Ans: A. L-gulonolactone oxidase
[Ref: Harper’s illustrated biochemistry, 30’h ed., pg. 200-201]

  • In liver, the uronic acid pathway catalyzes the conversion of glucose to glucuronic acid, ascorbic acid (except in human beings and other species for which ascorbate is a vitamin, vitamin C), andpentoses.
  • In human beings and other primates, as well as guinea pigs, bats, and some birds and fishes, ascorbic acid cannot be synthesized because of the absence of l-gulonolactone oxidase’

Question.2
Klenow fragment lacks the activity of?

AIIMS PG May-2018
A. 3′-5’exonuclease
B. 5′-3’exonuclease
C. 5′-3’DNA polymerase
D. 3′-5’DNA polymerase
Correct Ans: B
Explanation

Ans: B. 5′-3’exonuclease
Ref: Lehninger principles of biochemistry, vh ed, pg. 994 and Lippincott’s illustrated reviews 6th ed, pg. 74-75

  • DNA polymerase I is not the primary enzyme of replication; instead it performs a host of clean-up functions during replication, recombination, and repair.
  • When the 5′-3′ exonuclease domain is removed, the remaining fragment (Mr 68,000), the large fragment or Klenow fragment, retains the polymerization and proofreading activities.
  • Klenow fragment is a large protein fragment produced when DNA polymerase I from E. coli is enzymatically cleaved by the protease subtilisin.

Question.3
Which of the following techniques is based on RNA?

AIIMS PG May-2018
A. RT PCR
B. Sanger’s technique
C. Next generation sequencing
D. Western blot
Correct Ans: A
Explanation

Ans: A. RT PCR
[Ref: Harper’s illustrated biochemistry,3Oh ed, pg. 29,457 and Tietz Fundamental of clinical chemistry and molecular diagnostics, Vh ed.]
RT-PCR:

  • Reverse transcription polymerase chain reaction (RTPCR) is a laboratory technique combining reverse transcription of RNA into DNA (in this context called complementary DNA or cDNA) and amplification of specific DNA targets using polymerase chain reaction (PCR).
  • It is primarily used to measure the amount of a specific RNA. This is achieved by monitoring the amplification reaction using fluorescence, a technique called real-time PCR or quantitative PCR (qPCR). Combined RT-PCR and qPCR are routinely used for analysis of gene expression and quantification of viral RNA in research and clinical settings.
  • A method used to quantitate mRNA levels that rely upon a first step of cDNA copying of mRNAs catalysed by reverse transcriptase prior to PCR amplifi cation and quantitation.

 Sanger sequencing,

  • Also known as the chain termination method, is a technique for DNAsequencing based upon the selective incorporation of chain-terminating dideoxynucleotides (ddNTPs) by DNA polymerase during in vitro DNA replication.

Question.4
Which of the following does not require 5’capping?

AIIMS PG May-2018
A. tRNA of alanine
B. mRNA for histone
C. U6 snRNA
D. siRNA
Correct Ans: A
Explanation

Ans: A. tRNA of alanine
[Ref: Nature Reviews Molecalar Cell Biologt, volume 8, pg. 209-220 (2007) and Harper’s illustrated biochemistry 3Ah ed.]

  • Small nuclear RNAs contain unique 5′-caps.
  • mRNAs do have a 7-methylguanylate cap, abbreviated m7G.
  • tRNAsand rRNAs don’t require 5′ capping.
    • They have other modifications.

Question.5
Which of the following does not favor permissive euchromatin due to changes occurring at cytosine residues at CpG islands in DNA?

AIIMS PG May-2018
A. Methylation
B. Alkylation
C. Phosphorylation
D. Sumoylation
Correct Ans: A
Explanation

Ans: A. Methylation
[ref: Harper’s illustruted biochemistry, 3Ah ed., pg. 560.]

  • Methylation of cpG sites in the promoter of a gene may inhibit gene expression.
  • There is also evidence that low folate status results in impaired methylation of cpG islands in DNA, which is a factor in the development of colorectal and other cancers.

Question.6
In a reaction substrate is available in a concentration that is l000times the Km value of the enzyme. After 9 minutes of reaction, l7o substrate is converted to product (12 microgram/ml). If the concentration of the enzyme is changed to 1/3 and concentration of substrate is doubled. What is the time taken to convert the substrate into the same amount of product, i.e. I 2microgram/ml? 

AIIMS PG May-2018
A. 9 minutes
B. 4.5 minutes
C. 27 minutes
D. 13.5 minutes
Correct Ans: C
Explanation

Ans: C. 27 minutes
[Ref: Lehninger principles of biochemistry, 6t’ ed., pg. 204]

  • During derivation of Michaelis-Menten equation,
  • If total enzyme concentration is reduced by 1/3 and [S] is very high then Vmax also become l/3.
  • So in this question same amount of product will be fbrmed in 27 rninutes (3 x 9min)

Question.7
A Middle aged woman presents with fissures in mouth, tingling sensation and peripheral neuropathy. Investigations showed reduced glutathione reductase activity. which vitamin deficiency is the likely cause of this?

AIIMS PG May-2018
A. Vitamin B1
B. Vitamin 82
C. Vitamin 86
D. Vitamin B l2
Correct Ans: B
Explanation

Ans: B. Vitamin 82
[Ref: Harper’s illustrated biochemistry, 3Oh ed., pg. 556]

  • Deficiency of riboflavin (vitamin B2) is characterized by cheilosis, desquamation and inflammation of the tongue, and a seborrheic dermatitis.

Question.8
Fe absorption increases by which vitamin?

AIIMS PG May-2018
A. VitaminA
B. Vitamin C
C. Thiamine
D. Riboflavin
Correct Ans: B
Explanation

Ans: B. Vitamin C
[Ref: Harper’s illustrated biochemistry, 3Ah ed., pg. 541]

  • Inorganic iron is absorbed in the Fe2+ (reduced) state, and hence, the presence of reducing agents enhances absorption.
  • The most effective compound is vitamin C, and while intakes of 40 to 80 mg of vitamin C per day are more than adequate to meet requirements, an intake of 25 to 50 mg per meal enhances iron absorption, especially when iron salts are used to treat iron deficiency anemia.

Question.9
Which of the following enzyme activity decreases in fasting?

AIIMS PG May-2018
A. Hormone sensitive lipase
B. Glycogen phosphorylase
C. Acetyl CoA Carboxylase
D. Phosphofructokinase I
Correct Ans: D
Explanation

Ans: D. Phosphofructokinase I
[Ref: Harper’s illustrated biochemistry,3a,ed., pg. I88 and Lippincott’s illustratet! reviewsr 6’t’ ed., pg. 107]


Question.10

An adolescent male patient came with pain in calf muscles on exercise. On biopsy excessive amount of glycogen present was found to be present in the muscle. What is the most likely enzyme deficiency?

AIIMS PG May-2018
A. Muscle debranching enzyme
B. Phosphoftuctokinase I
C. Glucose-6 phosphatase
D. Phosphorylase enzyme
Correct Ans: D
Explanation

Ans: D. Phosphorylase enzyme
[Ref: Harper’s illustrated biochemistry, 3A” ed., pg. 178]

  • It is Mc Ardle disease.
  • Deficiency of muscle glycogen phosphorylase enzyme.
  • So the first step of glycogen breakdown will not occur.
  • Thus intact glycogen is seen in muscle biopsy.

Question.11
Fluoride released from fluoroacetate inhibits which metabolic pathway?

AIIMS PG May-2018
A. TCA cycle
B. Glycolytic pathway
C. Oxidative phosphorylation
D. ETC
Correct Ans: A
Explanation

Ans: A. TCA cycle
[Ref Harper’s illustrated biochemistry, 3A” ed, pg. 162, I7l’133 ]

  • Read the question carefully, it says fluoride from fluoroacetate and not sodium fluoride.
  • The poison fluoroacetate is found in some of plants, and their consumption can be fatal to grazing animals.
  • Some fluorinated compounds used as anticancer agents and industrial chemicals (including pesticides) are metabolized to fluoroacetate.
  • It is toxic because fluoroacetyl-CoA condenses with oxaloacetate to form fluorocitrate, which inhibits aconitase, causing citrate to accumulate.

Question.12
Vitamin K in its coenzyme form is regenerated by which enzyme?

AIIMS PG May-2018
A. Glutathione reductase
B. Pyruvate Carboxylase
C. Dihydrofolate reductase
D. Epoxide reductase
Correct Ans: D
Explanation

Ans: D. Epoxide reductase
[Ref: Harper’s illustrated biochemistry, 3Ah ed., pg. 555]

  • Vitamin K is the cofactor for the carboxylation of glutamate residues in the postsynthetic modification of proteins to form the unusual amino acid y-carboxyglutamate (Gla).
  • Vitamin K epoxide is reduced to the quinine by a warfarin-sensitive reductase, and the  uinone is reduced to the active hydroquinone by either the same warfarin-sensitive reductase or a warfarin-insensitive quinone reductase.

Question.13
Identify the phenomenon shown below?

AIIMS PG May-2018
A. Translation
B. Transformation
C. Complementation
D. Conjugation
Correct Ans: C
Explanation

Ans: C. Complementation
[Ref Ninfab Fundamental laboratory approachesfor biochemistry und biotechnology pg. 355-356 and Harper’s illustrated biochemistry,3Oh ed., pg’ 456]

  • This is schematic representation of the Blue-white assay, used to screen for recombinant vectors.
    • The method is based on the principle of o-complementation of the B-galactosidase gene.
    • So phenomenon is complementation. 

Question.14
Best investigation for Metabolic disorders is?

AIIMS PG May-2018
A. Western blot
B. Tandem mass spectrometry
C. PCR
D. Gel electrophoresis
Correct Ans: B
Explanation

Ans: B. Tandem mass spectrometry
[Ref: Tietz Tertbook of clinical chemistry and molecular biologt, 5’h ed,, pg. 2052]
Diagnosis of patients with inborn errors of metabolism:

  • Relies on specific tests such as ion-exehange chromatography and liquid chromatography with tandem mass spectrometry (LC-MS/MS) for amino acid analysis, gas chromatography/mass spectrometry (GC/MS) for organic acid analysis, tandem mass spectrometry (MS/MS) with (LC-MS/MS) or without liquid chromatographic separation for acylcamitine profiles, and LC-MS/MS or GC/MS for acylglycine profiles

Question.15
An infant presented to the OPD iwith a history of vomiting and malnutrition. On investigation the Guthrie test was found to be positive. All are true regarding this disease except?

AIIMS PG May-2018
A. Due to PAH enzyme defect
B. White patch of hair due to tryptophan deficiency
C. Phenyl acetate positive in urine
D. Mental retardation is present
Correct Ans: B
Explanation

Ans: B. White patch of hair due to tryptophan deficiency
[Ref: DM Vasudevsn Textbook of Biochemistry for medical student, Vh ed, pg. 236-237. Devlin’s textbook of Biochemistry, Vh ed, pg. 769]

  • Deficiency ofphenyl alanine hydroxylase is the cause for this disease.
  • The genetic mutation may be such that either the enzyme is not synthesized, or a non-functional enzyme is synthesized.
  • The classical PKU child is mentally retarded with an Ie of 50.
  • About 200% inmates of lunatic asylum may have PKU.
  • Guthrie test is a rapid screening test.

Ferric chloride test:

  • Urine of the patient contains phenylketones about 500 3000 mg/day.
  • This could be detected by adding a drop of ferric chloride to the urine.
  • A transient blue-green colour is a positive test.

Question.16
Best method to neasure HbA1c?

AIIMS PG May-2018
A. lsoelectric focusing
B. Affinity chromatography
C. Ion exchange chromatography
D. Electrophoresis
Correct Ans: C
Explanation

Ans: C. Ion exchange chromatography
[Ref: Tietz T*tbook of clinical chemistry and molecular biologt, lh ed^, pg. 1443-1444]

  • Techniques based on charge differences (ion-exchange chromatography, HPLC, electrophoresis, and isoelectric focusing), structural differences (affinity chromatography and immunoassay), or chemical analysis (photometry and spechophotometry).

Microbiology

Question.1
Organism most likely associated with VAP?

AIIMS PG May-2018
A. Acinetobacter
B. Klebsiella
C. Clostridum
D. Mycobacterium TB
Correct Ans: A
Explanation

Ans. A. Acinetobacter
Ventilator-associated pneumonia (VAP) :

  • Study from AIIMS Surgicul Department and ICUs shows that the Acinetobacter is nowadays the most common and most specific cause for Ventilator Associated Pneumonia (VAP).

Question.2
All of the following are associated with HHV8 except?

AIIMS PG May-2018
A. Kaposi sarcoma
B. Primary effusion lymphoma
C. Castlemann disease
D. T-cell leukemia
Correct Ans: D
Explanation

Ans. D. T-cell leukemia

  • Human Herpes Virus-8 Aka Kaposi’s sarcoma-associated herpes virus (KSHV).
  • Manifestations: In immunocompromised individuals (e.g., HIV), HHV-8 causes:
  • Kaposi sarcoma
  • Primary effusion lymphoma (body cavity based lymphomas)
  • Castleman’s disease (lymphoproliferative disorder of B-cells)

Question.3
Incubation period of LGV is?

AIIMS PG May-2018
A. 3-7 days
B. 7-10 days
C. 10-30 days
D. 30-90 days
Correct Ans: C
Explanation

Ans. C. 10-30 days

Feature

Syphilis

Herpes

Chancroid

LGV

Donovanosis

Incu-bation period

9-90 days

2-7 days

1-14 days

3 days-6

weeks

1-4 weeks (upto 6 months)


Question.4
(?d) T cell is associated with?

AIIMS PG May-2018
A. CD4
B. CD5
C. Cd8
D. First line defense against bacterial peptides
Correct Ans: D
Explanation

Ans. D-First line defense against bacterial peptides
y6 T cell:

  • (?d) T cell: gamma delta (?d) T cells have a TCR that is made up of one ? (gamma) chain and one d (delta) chain. This group of T cells is usually much less common than aß T cells, but are at their highest abundance in the gut mucosa, within a population of lymphocytes known as intraepithelial lymphocytes (IELs).
  • Constitute 5% of total r-cells, express y/6 chains of TCR chains; instead of ?/ß chains..
  • They lack both CD4 and CD8 molecules.
  • They do not require antigen processing and MHC presentation of peptides.
  • They lack both CD4 and CD8 molecules.
  • They do not require antigen processing and MHC presentation of peptides.
  • They are part of innate immunity as the(?d) receptors exhibit limited diversity for the antigen.
  • They are usually found in the gut mucosa, as intraepithelial lymphocytes (IELs).
  • The function of (?d) T-cells is not know, they may encounter the lipid antigens that enter through the intestinal mucosa.

Question.5
STERRAD is a gas plasma sterilizer used in operation theaters for disinfecting OT articles. Which of the following is the active agent used in it?

AIIMS PG May-2018
A. H2O2
B. N2O
C. Ozone
D. EtO
Correct Ans: A
Explanation

Ans. A. H2O2

  • The Sterrad Sterilization System by Advanced Sterilization Products (ASP) exploits the synergism between peroxide and low temperature gas plasma (an excited or ionized gas) to rapidly destroy microorganisms.

Question.6
In which of the following, viral load done by Real Time PCR is of no role in investigative procedures?

AIIMS PG May-2018
A. Person with hepatitis B on tenofovir therapy
B. HSV causing temporal encephalitis
C. BK virus in patient of allograft renal transplant
D. CMV PCR in blood of patient of liver transplant
Correct Ans: B
Explanation

Ans. B. HSV causing temporal encephalitis
[Ref: “Diagnosis ofherpesvirus infections ofthe central nervous system.’, Herpes : the journal of the IHMF I1 Suppl 2 2004, pp. 48A-564.]

  • The question simply asks in which of the given conditions calculation of Viral load is not required. In HSV causing temporal encephalitis the role of PCR is just to detect HSV DNA and make a diagnosis of the disease.
  • There is no role of detection of the viral load copies in the management or diagnosis of the disease.

Question.7
A 6-month child presented with diarrhea and vomiting for three days. Which of the following enterotoxin is most likely responsible for the condition?

AIIMS PG May-2018
A. NSP4
B. NSP6
C. VP3
D. VP7
Correct Ans: A
Explanation

Ans. A. NSP4

  • Rotavirus nonstractural protein 4 (NSP4) is known to function as an intracellalar receptor at the endoplssmic reticalam (ER) critical to virul morphogenesis and is the first characterized virul enterotoxin.
  • The associntion of NSP4 snd cuveolin-I contributes to NSP4 intrqcellular trfficking from the ER to the cell surfuce and speculate that exogenously added NSP4 stimulutes sigaaling molecules located in caveola microdomains.

Question.8
SD plasma destroys lipid enveloped virus. On SD plasma transfusion, which of the following infection is the likely possibility?

AIIMS PG May-2018
A. HIV
B. HAV
C. HBV
D. HCV
Correct Ans: B
Explanation

Ans. B. HAV

  • All the above given viruses except HAV are enveloped, so the HAV infection will spread even if the SD plasma destroys lipid envelope of the virus.

Pharmacology

Question.1
Which of the following act through G protein coupled receptors?

AIIMS PG May-2018
A. Ach Muscarinic receptors
B. Insulin receptors
C. Ach Nicotinic receptors
D. GABA-A receptors
Correct Ans: A
Explanation

Ans: A. Ach Muscarinic receptors
[Ref: Lippincott, 6’t’ed., Pg. 27-28]

  • Ml, M2, M3, M4 and M5 are Ach Muscarinic receptors.
  • They are G protein coupled receptors

Question.2
A 50-year-old male has fever for the past l-week with abdominal ilistention and loss of appetite. It is not responding to antibiotics and antimatarials. widal test is negative but RK39 dipstick test is positive. Which of the following drugs can be used in the above patient?

AIIMS PG May-2018
A. Bedaquiline
B. Linezolid
C. Fluconazole
D. Liposomal AmPhotericin B
Correct Ans: D
Explanation

Ans: D. Liposomal AmPhotericin B
[Ref Katzung, I3’h ed, Pg. 901-902]

  • RK39 dipstick test is a rapid immunochromatographic test being widely used in the diagnosis of visceral leishmaniasis.
  • Out of the given options Liposomal Amphotericin B is the only drug which is used to treat visceral leishmaniasis.

Question.3
Which of the following antifungal drug is developing drug resistance and has not been prescribed for tinea cruris and tinea corporis for the last 2 years?

AIIMS PG May-2018
A. Griseofulvin
B. Terbinafine
C. Itraconazole
D. Voriconazole
Correct Ans: A
Explanation

Ans: A. Griseofulvin
[Ref: Katzung, I3’t’ed., pg.832 and https://www.ncbi.nlm.nih.gov/pmc/articles/%5D

  • PMC4804599/
  • Among various options, topical terbinafine for 4 weeks appears to be the treatment of choice for lirnited disease (tinea corporis/cruris/pedis).
    • For more extensive disease, the choice is less clear.
  • It is also effective orally.
  • Both terbinafine (250 500 mg/day tbr 2 6 weeks) and itraconazole (100- 200 mglday for 24 weeks) appear to be effective.
  • Voriconazole is also an effective drug for these conditions.

Question.4
Drugs that should be given with prescription of registered medical practitioner only are included in which schedule ?

AIIMS PG May-2018
A. Schedule C
B. Schedule E
C. Schedule H
D. Schedule I
Correct Ans: C
Explanation

Ans. is ‘c’ i.e., Schedule H


Question.5

Which of the following gives the rate of drug absorption in plasma concentration graph?

AIIMS PG May-2018
A. Tmax and Cmax
B. Area under the curve
C. Tmax alone
D. Cmax alone
Correct Ans: A
Explanation

Ans: A. Tmax and Cmax
[Ref: Sharma & Sharma’s Principles of Pharmacolog, j’d ed., pg. 30-31 and hup:// whrw. e a r op e a n r ev i ew. o r g/wp /wp – c o n t e nt/ up I o a d s/6. p df]

  • From the plasma concentration graph, we obtain three important parameters
  • Cmax is the peak plasma concentration
  • Tmax i.e. time to attain the peak plasma concentration or Cmax
  • AUC (Area under the curve) of plasma concentration time graph
  • The 1st two parameters i.e. Cmax and the Tmax are the indicators of the rate of absorption.

Question.6
From which of the following routes, bioavailability of the drug is likely to be 100 percent?

AIIMS PG May-2018
A. Subcutaneous
B. Intravenous
C. Intramuscular
D. Intradermal
Correct Ans: B
Explanation

Ans: B. Intravenous
[Ref: Lippincott, 6th ed., pg. 3-4 and sharma & shatma’s Principles of Pharmacologt, 3’d ed., pg. 18-20]

  • lntravenous (iv) route, the drug enters the systemic circulation directly, bypassing the l” pass metabolism, so there is 100% bioavailability by this route.

Question.7
Hepatic First pass metabolism will be bypassed by the foltowing routes of drug adminisration excePt?

AIIMS PG May-2018
A. Oral
B. Intravenous
C. Sublingual
D. Intradermal
Correct Ans: A
Explanation

Ans: A. Oral
[Ref: Goodman & Gilman, I3’h ed., pg. 16-17 and sharma & sharms’s Principles of Pharmacologt, 3’d ed., pg. I8-20]

  • Oral route has certain disadvantages notably the 1st pass metabolism in liver before reaching the systemic circulation.

Question.8
Which of the following is wrongly.matched regarding mechanism of action ofanti-iungal drugs?

AIIMS PG May-2018
A. Azoles (fluconazole, itraconazole, miconazole): Inhibit lanosterol alpha demethylase thereby preventing ergosterol synthesis
B. Flucytosine: Inhibit microtubulesynthesis thus preventing mitosis
C. Echinocandins (caspofungin); act by inhibiting beta l,3 glucan synthesis
D. Amphotericin B binds with ergosterol resulting in disruption of cell membrane causing micropores and leakage of ions and cell death
Correct Ans: B
Explanation

Ans: B. Flucytosine: Inhibit microtubulesynthesis thus preventing mitosis
[Ref: Lippincott, 6h ed, pg. 535-541 and KatTung, l3th ed., pg. 926-g33]

  • Flucytosine is a pyrimidine anti-metabolite anti-fungal drug.
  • It enters the fungal cell via permease enzyme and then converted into compounds like 5-Fluorouracil, S-Fluorodeoxyuridine and 5′-monophosphate.
  • It then inhibits nucleic acid synthesis by inhibiting the enzyme thymidylate synthase.

Question.9
Latest drug reccntly approved for Amyotrophic Lateral sclerosis is?

AIIMS PG May-2018
A. Piracetam
B. Ceftriaxone
C. Edaravone
D. Doxycycline
Correct Ans: C
Explanation

Ans: C. Edaravone
[Ref https://www.ncbi.nlm.nih.gov/pmc/articles/PMC5737249/and https://www.br ai ntrop ic. c om/noo t rop ics/p i rac etam/]

  • Edaravone is a novel neuroprotective agent for the treatment of amyotrophic lateral sclerosis (ALS)

Question.10
Which of the following drugs is not used in Rheumatoid arthritis?

AIIMS PG May-2018
A. Etanercept
B. Leflunomide
C. Febuxostat
D. Methotrexate
Correct Ans: C
Explanation

Ans: C. Febuxostat
[Ref: Goodman antl Gilman, I3’1′ ed., pg. 704 antl Sharma & Sharma’s Principles o./ Pharmacology, 3’t ed., pg’ 383, 385-387]

  • Febuxostat is used for the treatment of chronic gout.
  • It is non-purine inhibitor (allopurinol is a purine analogue) of xanthine oxidase.
  • lt is more potent and selective than allopurinol.

Question.11
A patient requires 180 mg ceftriaxone. The vial contains 500 mg/5ml of ceftriaxone. You have a 2 ml syringe with l0 divisions per ml. How many divisions in the 2 ml syringe will you fill to give 180 mg ceftriaxone?

AIIMS PG May-2018
A. 9
B. 18
C. l0
D. 12
Correct Ans: B
Explanation

Ans: B. 18
500 mg/ 5 ml means 100 mg/l ml. Since 100 mg is present in 1 ml, therefore, 180 mg of drug is present in 1.8 ml.
Now it is clear that we must administer 1.8 ml of the above solution to administer the desired amount of drug.
Last thing is to calculate the divisions to administer 1.8 ml.
10 division per ml means each 0. 1 ml is equal to 1 division.
Therefore, 1 .8 ml will be equal to l8 divisions.


Question.12

Which of the following drugs is not used in typhoid fever?

AIIMS PG May-2018
A. Amikacin
B. Ciprofloxacin
C. Cefixime
D. Azithromycin
Correct Ans: A
Explanation

Ans: A. Amikacin
[Ref: Sharma & Sharma’s Principles of Pharmacologt, 3d ed., pg. 74t-749 and htrys://www.uplodate.com/contents/treatment-and-prevention-of-enteric-typhoid-und-paratyphoid-fever#HS]

  • Amikacin is a commonly used aminoglycoside.
  • Antimicrobial spectrum of aminoglycosides does not include salmonella.
  • It is used in gentamycin and tobramycin resistant infections.
  • It is used for organisms like Pseudomonas, Proteus, and Serratia. It is also effective in MDR-TB.

Question.13
A patient from,nqrth-eastern states was diagnosed to have infection with P. falciparum malaria. What is the most appropriate drug for this patient?

AIIMS PG May-2018
A. Artemether plus lumefantrine
B. Sulfadoxine plus pyrimethamine
C. Chloroquine
D. Mefloquine
Correct Ans: A
Explanation

Ans: A. Artemether plus lumefantrine
Ref: http://www.nvbdcp.gouin/Doc/Guidelines-Malaria-Diagnostic-Treatment-2014 pdf

  • Artemisinin Combination Therapy (ACT) should be given to all the confirmed P. falciparum cases.
  • The ACT recommended in the National Program all over India except northeastern states is artesunate (AS) daily for3 days and Sulfadoxine-pyrimethamine (SP) on Day 0.
  • Northeastern states presently recommended ACT in national drug policy is fixed dose combination of Artemether-lumefaritrine.
  • Hence, the clear-cut answer of this question is Artemether plus lumefantrine.

Question.14
Which of the following drug is commonly used in treatment for cancer associated thromboembotismt

AIIMS PG May-2018
A. Low molecular weight heparin
B. anti-thrombin III inhibitors
C. Direct Xainhibitors
D. Warfarin
Correct Ans: A
Explanation

Ans: A. Low molecular weight heparin

  • Current guideline-endorsed therapy options for cancer-associated thrombosis include low molecularweight heparin (LMWH), unfractionated heparin (UFH), warfarin & fondaparinux.
  • All current guidelines recommend LMWH for at least 3-6 months in cancer-associated VTE.

Question.15
Consider the following statements regarding typhus fever: a. It is caused by the rod-shaped spherical or pleomorphic gram-negative organisms which are smaller than true bacteria b. Erythematous maculopapular rash usually appear within first week in scrub typhus fever c. Weil-Felix test is positive in patients of typhus fever d. Tetracycline is the drug of choice Of these statements

AIIMS PG May-2018
A. 1 and 2 are correct
B. 2, 3 and 4 are correct
C. 1, 3 and 4 are correct
D. 1, 2, 3 and 4 are correct
Correct Ans: D
Explanation

Ans. D. 1, 2, 3 and 4 are correct
All the statements are correct and are self explanatory regarding typhus fever. Weil Felix test is not in common use now. Indirect immunofluorescence assay is used commonly. Doxycycline 200 mg/day is given for 7-15 days.


Question.16

Effect of Potassium ionophore valinomycin on beta cells of pancreas is?

AIIMS PG May-2018
A. Decrease the secretion of insulin
B. Insulin secretion increases
C. C-peptide secretion increases
D. Increase secretion of both insulin and C- peptide
Correct Ans: A
Explanation

Ans: A. Decrease the secretion of insulin
[Ref: https://www.sciencedirect.com/science/article/pii/0304416578903008 and http://www. ej e-o nline. o rg/co nte nt/8 8/I / I | 3]

  • Valinomycin interferes with the insulin releasing effect of glucose by increasing the potassium permeability of the B cell membrane.

Question.17
Which one of the following is not an adverse effect of salbutamol –

AIIMS PG May-2018
A. Tachycardia
B. Tolerance
C. Hypokalemia
D. Hypoglycemia
Correct Ans: D
Explanation

Ans. is ‘d’ i.e., Hypoglycemia
Inhaled ß-agonists have very few side effects. These are : (i) Muscle tremer & palpitation most common;(ii) hypokalemia; (iii) hyperglycemia; (iv) Tolerance; (v) Throat irritation; (vi) Ankle edema; and (vii) others (anxiety, headache, muscle cramps, dry mouth, arrhythmias , flushing & vasodilatation, hypoxemia, MI, sleep disturbance).


Question.18

Which drug causes flagellate pigmentation of skin?

AIIMS PG May-2018
A. Bleomycin
B. Minocycline
C. Vincristine
D. Daunorubicin
Correct Ans: A
Explanation

Ans: A. Bleomycin
[Ref: Goodman and Gilman, I3,h ed., pg. 1193]

  • Few case reports of bleomycin induced flagellate dermatitis and pigmentations are available in literature.

Question.19
Which of the following drug is commonly used for community acquired pneumonia in OPD?

AIIMS PG May-2018
A. Vancomycin
B. Ceftriaxone
C. Azithromycin
D. Streptomycin
Correct Ans: C
Explanation

Ans: C. Azithromycin
[Ref: https://www.uptodate.com/contents/trcatment-of-community-aquired-p n e u m o n ia- i n- a d u I t s -in -t h e – o u tp atie n t – s e tt in g# H 4]

  • Only one drug which is active orally i.e. Azithromycin.
  • We require OPD based treatment; hence Azithromycin is the best answer here.

Question.20
A medical student presented to the ED with protracted vomiting. For this he was given an antiemetic following which he developed abnormal posturing. Which of the following is the most likely drug to be given?

AIIMS PG May-2018
A. Metoclopramide
B. Ondansetron
C. Domperidone
D. Dexamethasone
Correct Ans: A
Explanation

Ans: A. Metoclopramide
[Ref: Katzung, I3’h ed, pg. 1070 md Lippincott, 6’t’ ed, pg. 408]

  • This is a typical case of metoclopramide induced extrapyramidal side effect (EPS) in the form of DYSTONIAS.
  • Metoclopramide crosses BBB and blocks central D2 receptors.
  • The major side effects of metoclopramide include EPS, such as those seen with the phenothiazine antipsychotics

Question.21
Manufaeturer of a drug company labels that the drug contains 500 mg of paracetamol. On analysis’ if was found to contain only 200 mg of drug. Which type of drug it is known as?

AIIMS PG May-2018
A. Counterfeit drug
B. Adulterant drug
C. Spurious drug
D. Unethical drug
Correct Ans: A
Explanation

Ans: A. Counterfeit drug
Ref:https://www.fda.gov/Drugs/ResourcesForYou/Consamets/BuyingUsingMedicineSafely/CounterfeitMedicine/ and hxp://www.globalforumljdorg/sites/default/files/docs/tibrary/lllflO-Evolution%20of/o20Definitions%20Definition.pdfand http://www.cdsco.nic.in/writereaddata/Gaidelines%20under%20new9620penal%20provi-sions9.pdf

  • According to the US FDA Counterfeit medicine is fake medicine.
  • It may be contaminated or contain the wrong or no active ingredient.
  • They could have the right active ingredient but at the wrong dose.
  • Counterfeit drugs are illegal and may be harmful to your health.

Question.22
Stoppage of which of the following drugs can result in anxiety and insomnia causing discontinuation syndrome?

AIIMS PG May-2018
A. Venlafaxine
B. ImiPramine
C. Valproate
D. OlanzaPine
Correct Ans: A
Explanation

Ans: A. Venlafaxine
Ref: Katzung, I3’h ed., pg. 524-525 and Goodman & Gilman, l3’h ed-, pg. 27372.

  • Sudden withdrawal of Shorter acting antidepressants particularly SSRIs (paroxetine and sertraline) and SNRI (venlafaxine) can lead to discontinuation syndrome and troublesome side effects.
  • These symptoms may include dizziness, headache, nervousness, nausea, anxiety and insomnia.
  • Out of the options provided Venlafaxine is the best answer.

Question.23
Heparin acts via which of the following adjuvants?

AIIMS PG May-2018
A. Antithrombin III
B. Protein C
C. Protein S
D. Thrombomodulin
Correct Ans: A
Explanation

Ans: A. Antithrombin III
[Ref: Goodman & Gilman, 13th ed, pg. 588]

  • Heparin binds to antithrombin III (AT-lll) and accelerate the rate at which it inhibits various coagulation proteases.

Question.24
Therapeutic index of a drug signifies:

AIIMS PG May-2018
A. Dose which produces maximum effect
B. Safety margin
C. Efficacy
D. Maximum response that can be elicited by a drug
Correct Ans: B
Explanation

Ans. B: Safety Margin

The therapeutic index/therapeutic ratio is a comparison of the amount of a therapeutic agent that causes the therapeutic effect to the amount that causes death.

It signifies safety margin.

Quantitatively, it is the ratio given by the lethal dose divided by the therapeutic dose.

A therapeutic index is the lethal dose of a drug for 50% of the population (LD50) divided by the minimum effective dose for 50% of the population (ED50).

A high therapeutic index is preferable to a low one: this corresponds to a situation in which one would have to take a much higher dose of a drug to reach the lethal threshold than the dose taken to elicit the therapeutic effect

Generally, a drug or other therapeutic agent with a narrow therapeutic range (i.e. with little difference between lethal and therapeutic doses) may have its dosage adjusted according to measurements of the actual blood levels achieved in the person taking it.


Question.25
Loading dose depends on ?

AIIMS PG May-2018
A. Volume of distribution
B. Elimination rate
C. Half life
D. Plasma volume
Correct Ans: A
Explanation

Ans. is ‘a’ i.e., Volume of distribution
Loading dose is governed by volume of distribution and volume of distribution is affected by lipid solubility.
Maintenance dose is governed by clearance (excretion) of drug and half life.

Forensic Medicine

Question.1

A 16-year-old girl comes to a doctor with fractured forearm. She told she tripped and fell but cigarette burns were observed on her forearm. What will be your Next step?

AIIMS PG May-2018
A. To infbrm higher authorities
B. To do a complete physical examination
C. To tell or discuss with colleagues that she is a case of abuse
D. To call local social worker for help
Correct Ans: B
Explanation

Ans: B. To do a complete physical examination

  • The first step after suspecting s case of abuse is to do a complete detailed history taking & physical examination of the patient to see sny other injuries which can confirm the findings.
  • Coming to conclusion just by seeing a burn which looks like cigarette burn on forearm can be misleading at times so a thorough examination of whole body is mandatory to confirm the above finding to child abuse.

Question.2
First hand knowledge” refers to

AIIMS PG May-2018
A. Opinion of a doctor in court
B. Handwriting expert
C. Common witness
D. Fingerprint expert
Correct Ans: C
Explanation

Ans: C. Common witness

  • “Except common witness option all other options are expert witnesses.
  • Only common witness can be considered as first-hand knowledge in court of law”.

Question.3
Match the following columns A. Heroin 1. Gunchi B. Spanish fly 2. Blister beetle C. Abrus 3. Blue rocket D. Aconite 4. Brown sugar

AIIMS PG May-2018
A. A-1 B-2 C-3 D-4
B. A-4 B-2 C-1 D-3
C. A-4 B-2 C-3 D-1
D. A-3 B-4 C-2 D-1
Correct Ans: B
Explanation

Ans: B. A-4 B-2 C-1 D-3
Cantharides (Spanish Fly)

  • Active principle: CcintharidinQ
  • Local application on skin: Vesicle or blister formation

Opium/Afim

  • Opium (poppy) is derived from Popover somniferum.
  • Toxic part: Unripe fruit capsule, latex juiceQ.

Question.4
Poisoning by which of these elements causes blind staggers alkali disease in livestock?

AIIMS PG May-2018
A. Magnesium
B. Manganese
C. Selenium
D. Molybdenum
Correct Ans: C
Explanation

Ans: C. Selenium
Prolonged oral exposure of cattle to elevated dietary selenium (Se) is associated historically with 2 clinical syndromes:

  • Alkali disease.
  • “Blind staggers”

Question.5
All are true about Doctrine ofborrowed servant except?

AIIMS PG May-2018
A. Hospital administration is responsible for the actions of interns and trainees
B. A nurse assisting a surgery is a borrowed servant ofthe hospital
C. Doctor assisting a surgeon for fee in OT
D. Physician is responsible for the acts ofthe orderlies and nursing staff done by his orders
Correct Ans: C
Explanation

Ans: C. Doctor assisting a surgeon for fee in OT
Borrowed servant doctrine:

  • The common law principle that the employer of a borrowed employee’ rather than the emp|oyee’s regular employer is liable for employees actions that occur while the emptoyee is under the control of the temporary employer.
  • Sometimes referred to as borrowed employee doctrine.

Question.6
A 14 year old female was claimed to be kidnapped, on interrogation she claims that she is not a minor and left home with the person on her own will. Court ordered for her age estimation. Given below are the X-rays of pelvis, wrist and bilateral elbow. What is her most probable age based on these?


AIIMS PG May-2018
A. 14 years
B. l6-17 years
C. l7-19 years
D. 2l-22 years
Correct Ans: D
Explanation

Ans: D. 2l-22 years

  • In figure 1, all the ossification centres are fused around elbow, so the age must be> 16 years.
  • In figure 2, lower end of radius and ulna are fused, so the age must be atleast 18 years.
  • In figure 3, Ischial tuberosity isfused, so the age must be around 20-22 years.
  • So the best answer for this question would be option (d) 21-22 years

Question.7
Calculate the Age of fetus having CRL 23 cm, foot length 7 cm. Talus is present and testes are present at superficial inguinal ring?

AIIMS PG May-2018
A. 5 months
B. 6 months
C. 7 months
D. 8 months
Correct Ans: C
Explanation

Ans: C. 7 months

  • Gestational age can be determined fiom maturation of chorionic villi; foot length and ossification centres.

Seventh month:

  • Length 35 cm; crown-rump length 23 cm; foot length 8 cm; weight 900 to 1200 g.
  • Nails are thick.
  • Eyelids open and pupillary membrane disappears.
  • Skin is dusky-red, thick and fibrous.
  • Meconium is found in the entire large intestine.
  • Testes are found at external inguinal ring.
  • Gallbladder contains bile and caecum is seen in the right iliac fossa.
  • Ossification centre is present in the talus.

Question.8
All are true about Doctrine ofborrowed servant except?

AIIMS PG May-2018
A. Hospital administration is responsible for the actions of interns and trainees
B. A nurse assisting a surgery is a borrowed servant ofthe hospital
C. Doctor assisting a surgeon for fee in OT
D. Physician is responsible for the acts ofthe orderlies and nursing staff done by his orders
Correct Ans: C
Explanation

Ans. C. Doctor assisting a surgeon for fee in OT
Borrowed servant doctrine:

  • The common law principle that the employer of a borrowed employee’ rather than the emp|oyee’s regular employer is liable for employees actions that occur while the emptoyee is under the control of the temporary employer.
  • Sometimes referred to as borrowed employee doctrine.

Question.9
All are the features present in male skull as compared to female skull except

AIIMS PG May-2018
A. Larger teeth
B. Larger foramen
C. Larger frontal sinuses
D. Larger fronta eminence
Correct Ans: D
Explanation

Ans: D. Larger fronta eminence

Feature Male Female
Architecture Rugged Smooth
Frontal eminence Small Large°
Parietal eminence Small Large°
Orbits Square° with smooth margin Rounded with sharp margin
Fore head Steeper° Vertical
Palate Large, broader and U shaped Smaller and parabola
Occipital condyle Large Small
Glabella More pronounced° Less pronounced
Fronto-nasal junction Distinct and angulated Smooth
Supraorbital ridges Prominent° Less prominent
Diagastric groove More deep Less deep
Mastoid process Large and blunt Small and pointed
Zygomatic arch More prominent Less prominent
Occipital protuberance Well marked Less marked

Pathology

Question.1
Abnormality in elastin protein can lead to all except ?

AIIMS PG May-2018
A. Fractures
B. Joint laxity
C. Aortic aneurysm
D. Subluxation of lens
Correct Ans: A
Explanation

Ans. A. Fractures
[Ref Clinical Outcomes of Elastin Fibre DeJbcts, J Cytol Histol 201 3, 4: I]

  • Abnormality in elastin protein can lead to abnormality in nrany systern.
  • It can lead to aortic aneurysm, joint laxity and subluxation of lens.

Question.2
Which of the following malignancy is least commonly associated with lymphatic spread ?

AIIMS PG May-2018
A. Basal Cell Carcinoma
B. Squamous cell Carcinoma
C. Malignant melanoma
D. Merkel cell Carcinoma
Correct Ans: A
Explanation

Ans. A. Basal Cell Carcinoma
[Ref: Robbins & Cotran, 9” ed., pg. I155-l I 57]

  • Basal cell carcinoma is a locally aggressive cutaneous tumor.
  • The rest of three tumors in the options have more tendency for lymphatic spread compared to BCC.

Question.3
Which of the following is the correct sequence of cellular events of acute inflammation?

AIIMS PG May-2018
A. Rolling ? Stable adhesion ? Activation of integrins ? migration via endothelium.
B. Stable adhesion ? Activation of integrins? Rolling ?-migration viaendothelium
C. Stable adhesion ? Rolling- ? Activation of integrins ? migration via endothelium
D. Activation of integrins ? migration via endotheliurn ? stable adhesion ? Rolling
Correct Ans: B
Explanation

Ans. B. Stable adhesion ?  Activation of integrins? Rolling ?-migration viaendothelium
[Ref: Robbins & Cotran, th ed., pg. 74, 75]
Acute inflammation process involved are:
In the lumen-

  • Margination,
  • Rolling,
  • Activation of Integrin and
  • Adhesion to endothelium

Question.4
A 5-years old child was presented with proptosis. Microscopic examination of the mass revealed a round cell tumor positive for Desmin immunohistochemical marker. Most likely diagnosis is ?

AIIMS PG May-2018
A. Leukemia
B. Embryonal rhabdomyosarcoma
C. Lymphoma
D. Primitive Neuroectodermal Tumor (PNET)
Correct Ans: B
Explanation

Ans. B. Embryonal rhabdomyosarcoma
[Ref: Robbins & Cotran, lh ed., pg. 1220-1222]

  • ln the four options provided , all four show morphology of a round cell tumour.
  • However Desmin positivity is seen only in cases of rhabdomyosarcoma.
  • In Embryonal rhabdomyosarcoma, the tumor cells mimic skeletal muscle at various stages of embryogenesis and consist of sheets of both prirnitive round and spindled cells in a myxoid stroma.
  • Immunohistochemistry for desmin and Myogenin confirm the rhabdomyoblastic differentiation.

Question.5
Which of the following stain is used for Acidic mucin?

AIIMS PG May-2018
A. Alcian blue
B. PAS
C. Masson’s trichrome stain
D. PTAH
Correct Ans: A
Explanation

Ans. A. Alcian blue
[Ref: Quick Reference Handbookfor Surgical Pathologists,20II, pg. 69-74]

  • Alcian blue stain is used for acidic mucin.
  • Acid mucins are present in goblet cells and esophageal submucosal glands.
  • They are Alcian Blue (AB.-positive (blue color).

Question.6
Which among the following laboratory investigation is best to reveal bleeding in Disseminated Intravascular Coagulation (DtC.?

AIIMS PG May-2018
A. Increased PT
B. Increased aPTT
C. Decreased fibrinogen
D. Increased FDPs
Correct Ans: D
Explanation

Ans. D. Increased FDPs
[Ref: Essentials of Haematology by Shirish Kawathalkar, 2″d ed., pg. 442-147]

  • Two types of DIC are acute (decompensated and chronic cornpensated).

Acute DIC:

  • Findings in acute DIC are low platelet, prolonged PT & APTT, low fibrinogen, and increased FDP and D-dimer.

Chronic DlC:

  • Findings are normal platelet count, normal PT & APTT.
  • However, FDP and D-dimer are increased.
  • So in acute DIC, all the four options are correct, while if we consider the question as chronic DIC, then best possible answer is increased FDP.

Question.7
Biopsy from an eight-year-old child with leg swelling showed smallround blue tumor cells consistent with diagnosis of Ewing’s sarcoma. What will be the best method to detect translocation t(ll;22) in this malignancy?

AIIMS PG May-2018
A. Conventional karyotyping
B. Next generation sequencing
C. FISH
D. PCR
Correct Ans: C
Explanation

Ans. C. FISH

  • FISH is a molecular cytogenetic technique that uses fluorescent probes that bind to only those parts of the chromosome with a high degree of sequence complementarity.

Question.8
Which of the following is true regarding blood transfusion of packed RBC?

AIIMS PG May-2018
A. Should be starled within 4 hours ol’receiving it fiom blood bank
B. Should be completed within 4 hours of receiving from blood bank
C. Wait till the patient is stable then transfuse, irrespective of any timing
D. Should be completed within 6 hours of receiving from blood bank
Correct Ans: B
Explanation

Ans. B. Should be completed within 4 hours of receiving from blood bank
[Ref: Essentials of’Haematology hy Shirish Kawathalkar, 2″d ed., pg. 487-4Bg.]

  • Whole blood after storing at 2-6 C loses functionally active platelets, Factor V and Factor VIII.
  • It has shelf lif’e of 35 days (CPDA- I ).
  • Transfusion of blood should be commenced within 30 minutes after removing it fiom refrigerator and should be completed within 4 hours of starting transfusion.
  • Indication for transfusing whole blood is fbr correction of both hypovolemia and red cell mass, such as acute massive blood loss.

Question.9
Storage temperature of RBC, Platelet, and Fresh Frozen Plasma (FFP) are:

AIIMS PG May-2018
A. RBC 2-6’C, Platelet 20-22″C, FFP -30’C
B. RBC -30’C, FFP 2-6″C, Platelet 20-22″C
C. RBC 20-22″C, Platelet 2-6″C, FFP -30″C
D. RBC 20-22″C, FFP -30’C, PLATELET 2-6’C
Correct Ans: A
Explanation

Ans. A. RBC 2-6’C, Platelet 20-22″C, FFP -30’C
[Ref: Essentials of Haematologlt by Shirish Kawathalkar, 2″u ed]

  • Whole blood and packed RBC is stored at 2-6 “C.
  • One unit of whole blood increase hemoglobin by l gm/dl and Hematocritby 3%.
  • Platelet is stored at20-24″C with shelf life of 5 days.
  • Fresh frozen plasma (200m1) is collected from single donor and frozen within 6 hours from collection.
  • Stored at -30 oC and has shelf life of 1 year.

Question.10
Which of the following is not a provisional entity as per WHO 2016 classification of Acute leukemia?

AIIMS PG May-2018
A. AML with hyperploidy
B. B-ALL with BCR-ABL like mutation
C. AML with BCR-ABL
D. Early T-cell precursor leukemia/Lymphoma
Correct Ans: A
Explanation

Ans. A. AML with hyperploidy
[Ref: ,yHO Classification of Tumours of Haematopoetic and lymphoid tissue, Revised fourth ed. 2017]

  • The classification of Hematolymphoid neoplasms was revised tn2016.

In this revision, followingprovisional entities were introduced:

  1. Myeloproliferative neoplasms (MPN): Myeloid/lymphoid neoplasms with PCMI-JAK2
  2. Myelodysplastic syndromes (MDS): Refractory cytopenia of childhood
  3. Acute myeloid leukemia (AML) and related neoplasms: AMI- with BCR-ABLl
  4. Acute myeloid leukemia (AML) and related neoplasms: AML with mu- tated RLINXl
  5. B-lymphoblastic leukemia / lymphoma: B-lymphoblastic leukemia/lym- phoma, BCR-ABLI-like
  6. B-ymphoblastic leukemia/lymphoma: B-lymphoblastic leukemia/lym- phoma with iAMP2l
  7. T-lymphoblastic leukemia/lymphoma: Early T-cell precursor lympho- blastic leukemia
  8. T-lymphoblastic leukemia/lymphoma: Natural killer (NK) cell lympho- blastic leukemia/lymphoma

ENT

Question.1
Gelles test is done in?

AIIMS PG May-2018
A. Otosclerosis
B. Serous otitis media
C. Traumatic hearing loss
D. Age related hearing loss
Correct Ans: A
Explanation

Ans: A. Otosclerosis
[Ref Dhingra Text book of ENT, 6″ ed., ch-4]
Gelle’s test:

  • Test of bone conduction and examines the effect of increased air pressure in ear canal on the hearing.
  • It was a popular test to find out stapes fixation in otosclerosis but has now been superceded by tympanometry.

Question.2
Best Surgery to relieve intractable Vertigo in a menieres disease patient is?

AIIMS PG May-2018
A. Labyinthectomy
B. Cochlear Neurectomy
C. Endolymphatic sac decompression
D. Cochleosacculotomy
Correct Ans: A
Explanation

Ans: A. Labyinthectomy
(Ref Glasscock-Shambaugh Surgery of the Ear 6′ ed., pg. 573).

  • Labyrinthectomy is the most destructive procedure in the treatment of Meniere’s as it destroys both hearing and vestibular function.

Question.3
A 12 year adolescent male presents with unilateral mass in nose with recurrent spontaneous bleeding. On examination a pink mass covered with mucosa is found in the nasopharynx. Which of the following is not to be done?

AIIMS PG May-2018
A. Biopsy is done for diagnosis
B. CECT is done
C. Endoscopy surgery can be done
D. Angioembolization can be performed
Correct Ans: A
Explanation

Ans: A. Biopsy is done for diagnosis
[Ref: Scotl Brown, Vt’ ed., pg. 2438]

  • Nowadays, thediagnosis is based on the CT and MR appearances that are sometimes confirmed by angiography.
  • A trans-nasalbiopsy is not necessary and can provoke brisk haemorrhage.

Question.4
Caloric test was done on right side with cold water. Eyes moved to the opposite side. Which of the following corresponds to correct interpretation of nystagmus in this test?

AIIMS PG May-2018
A. Eyes move slowly to right
B. Eyes move rapidly to left
C. Eyes move slowly to left
D. Eyes move rapidly to right
Correct Ans: C
Explanation

Ans: C. Eyes move slowly to left
[Ref: Glasscock-Shambaugh Surgery ofthe Ear 6’h ed., pg. 124]

  • Endolymph will flow toward the ampulla of the horizontal canal (resulting in an increase in afferent discharge rate) for warm irrigations and away from the ampulla (resulting in a decrease in afferent discharge rate) for cold irrigations.

Question.5
A 55 year old male presents with a lesion of 2.5*3.5 cm on lateral border of tongue with induration. There is no associated lymphadenopathy. What is the staging according to the 8th AJCC guidelines?

AIIMS PG May-2018
A. T2N0M0
B. T3N2M0
C. T4N3M0
D. T3N2M0
Correct Ans: A
Explanation

Ans: A. T2N0M0
[Ref: AJCC Cancer staging munual I’h ed., ch-7, pg. 79,90]
Cancer of the lip and oral cavity TNM staging AJCC UICC 2017

Ophthalmology

Question.1

Which of the following is the nucleus for upwards gaze?

AIIMS PG May-2018
A. Paramedian Pontine Reticular Formation
B. Nucleus Raphe Magnus
C. Cuneiform nucleus
D. Nucleus of Cajal
Correct Ans: D
Explanation

Ans: D. Nucleus of Cajal
[Ref: Conjugate Gaze Palsies – Neurologic Disorders.,, Merck Manuals professional, Ed,,2015 and Merck Sharp & Dohme Corp, I Mar. 2014. lleb. 8 Oct. 2015].

  • The three key structures in the control of the vertical gaze includes,
  • Rostral interstitial nucleus of medial longitudinal fasciculus (riMLF).
  • Interstitial nucleus of Cajal (INC)
  • Posterior commissure (PC).

Question.2
Identify the pathology in the below histopathology picture of retina?

AIIMS PG May-2018
A. Drusen
B. Phagosome
C. Basal laminar deposit
D. Basal linear deposit
Correct Ans: A
Explanation

Ans: A. Drusen
[Ref: American Academy of Ophthalmologt, 20 t 7]

  • Above retinal histopathology image is of Drusen characteristic feature of ARMD i.e. Age related macular degeneration.
  • Drusen are a hallmark of ARMD.
  • On histopathology, nodular drusen appear as eosinophilic dome-shaped structures situated between Bruch’s membrane and the displaced, attenuated RPE overlying the drusen.

Question.3
Stenopic slit’is used for all except-

AIIMS PG May-2018
A. Fincham’s test
B. Determine the axis of cylinder
C. Corneal tattooing
D. lridectomy
Correct Ans: C
Explanation

Ans: C. Corneal tattooing
[Ref: A, K. Khurana Comprehensive Ophthalmologt,4,h ed]

  • Stenopic slit is useful in finding out the axis of the cylinder.
  • (option b) stenopic slit is used in cases of corneal opacities to find out the optimal site for optical iridectomy.
  • After dilating the pupil with a mydriatic, the slit is rotated in front of the eye and the axis which gives maximum clarity is chosen for optical iridectomy

Question.4
Which of the following is used as an adjuvant therapy for fungal corneal ulcer?

AIIMS PG May-2018
A. Atropine eye drops
B. Pilocarpine eye drops
C. Dexamethasone
D. Lidocaine
Correct Ans: A
Explanation

Ans: A. Atropine eye drops

  • Treatment usually includes antifungal medications depending on sensitivity.
  • Atropine eye drops are given to release ciliary spasms & to give symptomatic reliel.

Question.5
A 55-year-old man complaints of glare in eye while driving his car in the night, his best corrected vision is 6112. Whtt is the most probable diagnosis?

AIIMS PG May-2018
A. ARMD
B. Post capsular opacification lens
C. Corneal degeneration
D. Diabetic retinopathy
Correct Ans: B
Explanation

Ans: B. Post capsular opacification lens

  • Posterior capsular opacification of lens is responsible for glare in eye at night.
  • At night pupil dilates usually & hence more light enters in the eye, so more light creates more aberrations in the eye, hence more glare.
  • In early posterior subcapsular cataract visual acuity canbe 6/12

Question.6
A 15-year-oldboy presented with headache and blurring of vision. On examination there was diptopia on looking towards left in the right eye. What is your diagnosis?

AIIMS PG May-2018
A. Tb meningitis
B. Internuclear ophthalmoplegia
C. Cranial neuritis
D. Demyelination
Correct Ans: B
Explanation

Ans: B. Internuclear ophthalmoplegia
Demyelination
Ref Bilateral Internuclear ophthalmoplegio in Multiple sclerosis,iejm journal,20l6
Typically, symptoms & examination findings i.e. diplopia on looking towards left in the right eye is characteristic of internuclear ophthalmoplegia. 


Question.7

In 3rd nerve palsy all seen except?

AIIMS PG May-2018
A. Pupil dilation
B. Ptosis
C. Outward upward rolling of pupil
D. Impaired pupillary reflex
Correct Ans: A
Explanation

Ans: A. Pupil dilation
Ref: American Academy of Ophthalmolog 2018, third nerve palsy

  • The pathway for pupillary constriction for each eye has an afferent limb taking sensory information to the midbrain, and two efferent limbs (one to each eye).

Preventive & Social Medicine

Question.1
You went to a sub center as part of an audit. How many infants should be registered with a health worker working there?

AIIMS PG May-2018
A. 50
B. 100
C. 150
D. 200
Correct Ans: B
Explanation

Ans: B. 100
Ref: K Park, 23d ed., pg. 524, 567

  • Estimated no. of infants in a health sub centre covering 5000 population can be calculated as follows:
  • No. of live birth = Birth rate x Population/ 1000
    • = 20.4 x 5000/1000 = 102
  • Infant mortality for sub centre = No. of live birth x IMR/1000
    • = 102 x 34/1000
    • = 3.5 (for calculation purpose we assume it 4)
  • Therefore, total number of infants = 102 — 4
    • = 98
  • Hence correct answer is 100 (nearest value).

Question.2
Under National Health Mission which committee makes plan for village health?

AIIMS PG May-2018
A. Panchayat health committee (PHC)
B. Village health planning and management committee (VHPMC)
C. Village health sanitation and nutrition committee (VHSNC)
D. RogiKalyan Samiti
Correct Ans: C
Explanation

Ans: C. Village health sanitation and nutrition committee (VHSNC)
Ref: K Park, 23’d ed,, pg. 450-451 and Health Policies and Programmes in India, DK Taneja, I2’h ed., pg. 82
VHSNC:

  • Sub-committee or a standing committee of the Gram Panchayat.
  • The VHSNC acts as a platform for convergence between different departments & committees at village level.
  • The VHSNCs shall be supported to develop village health plans to ensure convergent action on social determinants of health, ensure access to health services, especially of the more marginalised sections in the village, and support the organization of the Village Health and Nutrition Day.

Question.3
All of the following can be analyzed with chi square test except?

AIIMS PG May-2018
A. Sex and stage ofcancer
B. Heart rate/min and Age
C. Benign or malignant, and type of surgery
D. Age group and cancer stage
Correct Ans: B
Explanation

Ans: B. Heart rate/min and Age
Ref: Biostatistics Principles and Practice by B Antonisamy, Mc graw hitt publication,pg. I17.

  • In given question, all options except heart rate/minute and age are of qualitative variables (nominal or ordinal).
  • Qualitative variables are usually presented in frequencies or count.
  • Chi square test is used to compare any difference or association between two qualitative variables.
  • Heart rate/minute and age are quantitative variables where chi square test cannot be applied.
  • Hence correct answer is b here.

Question.4
Which of the following parameters is used to determine the sensitivity of vaccine due to heat?

AIIMS PG May-2018
A. VVM
B. VMV
C. VCM
D. VMM
Correct Ans: A
Explanation

Ans: A. VVM
[Ref: K Park, 23’d ed, pg. 110]

  • VVM (Vaccine Vial Monitor)
  • VVM is a label containing a heat-sensitive material which is placed on a vaccine vial to register cumulative heat exposure over time.
  • It has outer blue circle and inner white square.

Skin

Question.1
A patient came with history of joint pain and fever for one week. For which the patient took NSAIDS foilowing which he developed rash and hyperpigmentation on nose. What is the most likelv diagnosis?

AIIMS PG May-2018
A. Dengue
B. fixed Drug eruption
C. Chikungunya
D. Melasma
Correct Ans: C
Explanation

Ans : C Chikungunya
[Ref: Hypapigmqtdion and &ilangurwfetEn &hasnw A” An has Demaor 20 I 6;9 I :g60-I r]

  • Chikungunya fever (cF) is an acute febrile illness presenting with symptoms like intense asthenia, arthralgia, myalgia and headache and is caused by chikungunya virus (CV).
  • Among the skin lesions, maculopapular rash is common, but seen in several viral illnesses, therefore, not useful in suspecting CF.
  • Hyperpigmentation is a unique feature noted in CF.
  • Hyperpigmentation associated with CF is macular and most commonly affects nose and cheeks.

Question.2
Which Nerve biopsy is taken to diagnose neuritic leprosy?

AIIMS PG May-2018
A. Median Nerve
B. Ulner Nerve
C. Radial Cutaneous Nerve
D. Sural nerve
Correct Ans: C
Explanation

Ans: C: Radial Cutaneous Nerve
[Ref: Kumar B, Dogra S. ch- 15, Case definition and clinical types of leprosy, pg. 259.IAL Tixtbook of leprosy. 2d ed,]

  • Ref: Ponnaiya J. Chapter 19: Laboratory diagnosh, pg. 285. AL Textbook of Leprosy,7d ed.,
  • Ref: Rao PN, Suneetha S. Parc neuritic leprosy: Cunent status and relevance Indian J Dermatol Venereol Leprol 2 0 I 6 ; I 2 : 2 5 2-6 I ).
  • Pure neuritic Hansen is characterised by an area of sensory loss along the distribution of an involved nerve trunk with or without motor deficit.
    • In the absence of any Skin patch.
  • The suitable nerves include supraorbital branch of the fifth cranial nerve, supra clavicular nerve, great auricular nerve in the neck, radial cutaneous nerve at the wrist, a cutaneous nerve ofthe forearm or thigh, sural nerve at the back of the leg or superficial peroneal nerve on the dorsum ofthe fbot.
  • The nerves usually chosen for biopsy are a branch of sural nerve at the level just above the ankle, or a branch ofradial cutaneous nerve at the wrist resion.

Question.3
Which component of cement causes allergic contact dermatitis??

AIIMS PG May-2018
A. Cobalt
B. Nickel
C. Iron
D. Chromium
Correct Ans: D
Explanation

Ans : D: Chromium

  • Most common allergen is hexavalent chromium salts.
  • Allergic contact dermatitis more than irritant contact dermatitis.
  • Level of exposure needed for allergic contact dermatitis 100-400 nglcm2.
  • Tartarate, glycine, silicone, dapsone, ferrous sulphate causes conversion from chromium (VI) to chromium (III), which is less allergenic.

Psychiatry

Question.1
Which of the following is not a feature of atypical depresslon? 

AIIMS PG May-2018
A. TCAs are better than MAO and SSRI for treatment
B. Variable Mood responding to positive stimulus in surroundings
C. Increased sleep and weight gain
D. Marked fatigue and heaviness in the body
Correct Ans: A
Explanation

Ans. A : TCAs are better than MAO and SSRI for treatment

(Ref: Complete review ofpsychiatry by Dn Prashant Agrawal, ed-, 201g, ch – 3, pg, 60)

Symptom Atypical
Mood reactivity Mood reactivity present (brightens in response to positive events)
Weight appetite Sleep Significant weight gain or increase in appetite Excessive sleep all through the day
Psychomotor activity Leaden paralysis (heavy feeling in arms, legs)
Personality thinking Interpersonal rejection sensitivity not limited to mood episodes
Diurnal variation Depression likely worse inevening (not part of diagnostic criteria)

Question.2
A person who is shy, emotionally cold, shunning close relationships, introspective and prefers solitary activities, resides alone. There are no delusions and hallucinations. Which personality disorder is he likely to be classified into?

AIIMS PG May-2018
A. Schizoid personality disorder
B. Paranoid personality disorder
C. Emotionally unstable personality disorder
D. Antisocial personality disorder
Correct Ans: A
Explanation

Ans. A: Schizoid personality disorder
[Ref Complete review of psychiatry by Dr. Prashant Agrawal, ed., 2018, ch – 14, pg. 280]
Personality disorders are organized into three “clusters” in both the DSM-IVTR and DSM-5.
The “cluster A” personality disorders include:

  • Schizotypal Personality Disorder
  • Paranoid Personality Disorder
  • Schizoid Personalitv Disorder 

Schizoid Personalitv Disorder:

  • 2 striking features.
  • Lack of interpersonal relationships and the lack ofdesire to obtain such relationships .
  • Presence of ‘negative symptoms’in the absence of psychotic- like cogritive and perceptual distortions.

Question.3
Which of the following is not an adverse effect of Escitalopram?

AIIMS PG May-2018
A. Nausea
B. Vivid dreams
C. Anorgasmia
D. Sialorrhoea
Correct Ans: D
Explanation

Ans: D: Sialorrhoea
[Ref: Complete review of psychiatry by Dr. Prashant Agrawal, ed., 2018, ch – 3, pg. 61]
SSRIs side effects:

  • Based on onset, duration and severity.
  • Nausea & giddiness – Early, mild, time-limited.
  • Sexual Dysfunction – All SSRIs.
  • Most common adverse effect on long-term treatment.
  • Anorgasmia, inhibited orgasm & decreased libido.
  • Most frequent are nausea, diarrhea, anorexia vomiting, flatulence, and dyspepsia.
  • Sertraline and fluvoxamine – Most intense GI symptoms.

Paroxetine:

  • Pronounced weight gain in young women.
  • Constipation – Anticholinergic activity.

Radiology

Anaesthesia

Question.1
A patient is undergoing MRND for laryngeal malignancy; while dissecting the venous tributaries the surgeon elevated the internal jugular vein for ligation. Suddenly the patients EtCO2 dropped from 3g mmHg to 12 mmHg and the patient developed hypotension along with cardiac arrhythmia. Which of the following is most likely cause??

AIIMS PG May-2018
A. Sympathetic overactivity
B. Vagal stimulation
C. Venous air embolism
D. Carotid body stimulation
Correct Ans: C
Explanation

Ans: C: Venous air embolism
[Ref Millers anesthesia &h ed-, pg. 2170]

  • Diagnosis standard of care is precordial doppler (left or right parastemal, between 2nd and 3rd ribs) + ETC02 monitoring although this is not the most sensitive test – TEE is most sensitive.
  • Pulmonary artery pressure will rise, and CO2 will fall alter VAE.

Internal Medicine

Question.1
A 22-year-old man presents with diarrhea and intolerance to dairy products, on investigation he was found to have lactase deficiency. Agent least likely to cause lactose intolerance among these is?

AIIMS PG May-2018
A. Condensed MILK
B. Skimmed Milk
C. Yoghurt
D. Ice cream
Correct Ans: D
Explanation

Ans: D. Ice cream


Question.2

A known 60-year-old male Diabetic and hypertensive patient was found unconscious in the morning. On examination pulse- 120, bp- 180/100 and there was positive extensor plantar. What is next step to be done?

AIIMS PG May-2018
A. Order CT scan
B. Immediately reduces bp with antihypertensives
C. Give I/v mannitol
D. Check blood glucose
Correct Ans: D
Explanation

Ans: D. Check blood glucose

  • Quick assessment of Blood sugar should be done to rule out hypo or hyperglycemia is needed.
  • In hypoglycemic coma, the eyeball tension is normal, there is profuse sweating and the plantar responses are extensor always either normal or subnormal in uncomplicated cases of either condition.
  • It is well to remember that although a comatose patient is a known diabetic, the diagnosis is not always one of ‘hyper- or hypo-glycaemia.

Question.3
40-year male patient who was a known smoker presents in AIIMS OPD with fever, latigue, jaundice, yellow colored urine with clay colored stools. For the past few days he had developed aversion to cigarette smoking. On examination scleral icterus was present. LFT are: Total Bill 18.5 Direct Bill 7.5 SGOT 723 SGPT 812 What investigations will you send for ruling out acute viral hepatitis?

AIIMS PG May-2018
A. HBsAg, IgM antiHBC, AntiHCV AntiHEV
B. AntiHAV, HBsAg, IgM antiHBc, AntiHCV
C. HBsAg, IgM antiHBC,AntiHDV AntiHCV AntiHEV
D. AntiHAV IgMantiHBC, AntiHCV, AntiHEV.
Correct Ans: B
Explanation

Ans: B. AntiHAV, HBsAg, IgM antiHBc, AntiHCV
Ref: Harrison, I8’t’ ed., pg. 328, 2549-2552, 2550t, 2551t

HB- sAg

IgM Anti- HAV

IgM Anti- HBc

Anti-HCV

Diagnostic Interpretation

+

 

+

 

Acute hepatitis B

 

 

 

 

Chronic hepatitis B

+

+

 

 

Acute hepatitis A superim? posed on chronic hepatitis B

+

+

+

 

Acute hepatitis A and B

+

 

 

Acute hepatitis A

+

+

 

Acute hepatitis A and B (HBsAg below detection threshold)

 

 

+

 

Acute hepatitis B (HBsAg below detection threshold)

 

 

 

 

Acute hepatitis C 


Question.4
A -wave in JVP indicates:  

AIIMS PG May-2018
A. Atrial relaxation
B. Atrial contraction
C. Bulging of tricuspid valve into right atrium
D. Ventricular contraction
Correct Ans: B
Explanation

Ans. B: Atrial contraction

Waves

  • a – presystolic; produced by right atrial contraction
  • c – bulging of tricuspid valve into the right atrium during ventricular systole (isovolumic phase)
  • v – occurs in late ventricular systole; increased blood in right atrium from venous return

Descents

  • x – combination of atrial relaxation, downward movement of the tricuspid valve and ventricular systole
  • y – tricuspid valve opens and blood flows in to the right ventricle 

Abnormal waveform causes

Dominant a wave

  • Pulmonary stenosis
  • Pulmonary hypertension
  • Tricuspid stenosis

Cannon a wave

  • Complete heart block
  • Paroxysmal nodal tachycardia

Ventricular tachycardia

Dominant v wave

  • Tricuspid regurgitation

Absent x descent

  • Atrial fibrillation

Exaggerated x descent

  • Cardiac tamponade
  • Constrictive pericarditis

Sharp y descent

  • Constrictive pericarditis
  • Tricuspid regurgitation

Slow y descent

  • Right atrial myxoma

Question.5
A Patient is having Ph- 7.12, HCO3- 28 and PCO2- S0mmhg. What is the acid base disorder in this patient?

AIIMS PG May-2018
A. Metabolic acidosis with respiratory compensation
B. Metabolic alkalosis with respiratory compensation
C. Respiratory acidosis with renal compensation
D. Respiratory alkalosis with renal compensation
Correct Ans: C
Explanation

Ans: C. Respiratory acidosis with renal compensation
pH-7.12 so its acidosis.
PCO2 is 5o mmHg (normal range 35-45 mmHg) which is higher than the normal s/o respiratory acidosis.
HCO3- is 28 mmol/L (18-24 mmol/L) which is higher than the normal range s/o metabolic compensated alkalosis.


Question.6

A Female patient was brought to the ER with altered sensorium. On examination BP was 88/67mm with a pulse of 60/min. Rectal temperature was 34 Celsius. There was associated history of constipation, dry skin and menorrhagia. What is the diagnosis?

AIIMS PG May-2018
A. Myxedema Coma
B. Septic Shock
C. Hypothermia
D. Cardiogenic Shock
Correct Ans: A
Explanation

Ans: A. Myxedema Coma
Ref: Harrison, I8’t’ ed., pg. 2922.

  • Myxedema coma is a state of decompensated hypothyroidism.
  • A person may have lab values identical to a “normal” hypothyroid state, but a stressful event (such as an infection, myocardial infarction or stroke) precipitates the myxedema coma state, usually in the elderly.
  • Primary symptolns of myxedema coma are altered rnental status and low body ternperature.
  • Low blood sugar, low blood pressure, hyponatrereria, hypercapnia, hypoxia, slowed heart rate, and hypoventilation may also occur. 

Question.7
An old tady developed intractable vomiting for 7 days with loss of fluid in stools, decrease food intake and developed altered sensorium. She is brought to the hospital, where fluid loss is corrected with rapid fluids and electrolyte correction is given for hyponatremia. During the treatment, the patient started having quadriplegia and mutism. What is the diagnosis?

AIIMS PG May-2018
A. A rare complication of Rapid and aggressive sodium correction
B. Common complication of Hypernatremia
C. Brain stem infarcts
D. None ofthe above
Correct Ans: A
Explanation

Ans: A. A rare complication of Rapid and aggressive sodium correction
[Ref Harrison, I8’t’ ed., pg. 347, 2259, 2260J]

  • Central pontine myelinolysis (CPM) or osmotic demyelination syndrome (ODS) is a neurological disorder caused by severe damage of the myelin sheath of nerve cells in the area of the brainstem termed the pons, predominately of iatrogenic, treatrnent-induced cause Rapid and aggressive sodium correction.
  • It is characterized by acute paralysis, dysphagia (difficulty swallowing), and dysarthria (difficulty speaking), and other neurological symptoms.
  • Na correction to be done in a slow manner.

Question.8
A healthy middle-aged man was arguing with his brother and got emotionally upset due to the arguments with his brother, he suddenly developed chest pain and collapsed. When brought to the hospital’ hewas declared dead.What is the diagnosis?

AIIMS PG May-2018
A. Takotsubo cardiomyopathy
B. Dilated cardiomyopathy
C. Arrthmogenic right ventricle dysplasia
D. Chronic ischemic cardiomyopathy
Correct Ans: A
Explanation

Ans: A. Takotsubo cardiomyopathy
[Ref Harrison, 18’t’ ed., pg, 1964]

  • Takotsubo cardiomyopathy, also known as stress cardiomyopathy, is a type of non-ischemic cardiomyopathy in which there is a sudden temporary weakening of the muscular portion of the heart.
    • This weakening may be triggered by emotional stress, such as the death of a loved one, a break-up, rejection from a partner or constant anxiety.
  • “Broken heart syndrome”.
  • Stress cardiomyopathy is now a well-recognized cause of acute heart failure, lethal ventricular arrhythmias, and ventricular rupture.

Question.9
A patient presented with headache and fever for 7 days. While doing a lumbar puncture, it was noticed that the opening CSF pressure increased on jugular vein compression and became normal on relieving the pressure on jugular vein. What is the interpretation of this?

AIIMS PG May-2018
A. Subdural blockage
B. Subarachnoid blockage
C. Arachnoid villi blockage
D. Patent subarachnoid space
Correct Ans: D
Explanation

Ans: D. Patent subarachnoid space
[Ref: Text book ofneurology (By Navneet), pg, 605]

  • The narrowed fspinal] channel impedes movement of fluid with an increase in pressure above the compression site.
  • The increment in pressure above the obstruction can be demonstrated by compression of the neck which produces an increase in venous blood in the cranial cavity, with concomitant reduction in space for the cerebrospinal fluid.
  • The increased fluid pressure immediately transnritted throughout the systern normally can be demonstrated with a. manometer attached to a lumbar puncture needle.
  • In lesions of the cord the anometric change is greatly retarded.
    • Referred as Queckenstedt’s maneuver.
  • Rise in opening pressure is suggestive of patent spinal canal.

Surgery

Question.1
A male came to the ER after car accident. He had dyspnea and chest pain with ecchymosis on anterior chest wall. On examination, pulse -120lmin, BP-80/50mmHg, breaths sounds were decreased on left side, JVP was raised and tympanic note was present on percussion. Pelvis and extremities were normal.What is your diagnosis?

AIIMS PG May-2018
A. Tension pneumothorax
B. Cardiac tamponade
C. Massive hemothorax
D. Hydropneumothorax
Correct Ans: A
Explanation

Ans. A.Tension pneumothorax
[Ref: Bailey and Love’s short practice ofSurgery, 26″‘ ed., pg. 354]

  • Beck’s triad describes symptoms commonly associated with tension pneumothorax.

The three components of Beck’s triad are:

  • Distended neck veins
  • Muffied heart sounds. and
  • Hypotension.
  • Other symptoms include tachypnoea, dyspnoea, tachycardia, and hypoxia.

Question.2
Whieh of the following is correct regarding the boundaries of triangle of doom?

AIIMS PG May-2018
A. Medially vas deferens, laterally gonadal vessel, inferiorly peritoneum
B. Laterally vas deferens, medially gonadal vessel, inferiorly peritoneum
C. Laterally medial umbilical ligament, medially gonadal vessel, Lateral inferiorly peritoneum
D. Laterally gonadal vessels and medially lateral umbilical ligament, inferiorly peritoneum
Correct Ans: A
Explanation

Ans. A. Medially vas deferens, laterally gonadal vessel, inferiorly peritoneum
[Ref: Master techniques in surgery Hernia, Josef E. Fischer, 2013, pg. l6l-162]
Boundaries of Triangle of Doom (Content – External iliac vessels)

  • Vas deferens medially
  • Spermatic vessels laterally
  • Peritoneal fold inferiorly

Boundaries of Triangle of Pain (Content – Lateral femoral cutaneous nerve)

  • Laterally iliopubic tract
  • Medially Gonadal vessels

Question.3
Which of the following is wrong regarding superior mesenteric artery syndrome?

AIIMS PG May-2018
A. Superior mesenteric artery is compressed by third part of duodenum at the ligament of Treitz attachment
B. Superior mesenteric artery has a normal angle between 38-65 degree in relation to duodenum
C. Strong procedure is corrective surgery in which ligament of Treitz is divided
D. Superior mesenteric artery syndrome is characterized by an angle less than 25 degree due to loss of intervening mesenteric pad of fat
Correct Ans: A
Explanation

Ans. A. Superior mesenteric artery is compressed by third part of duodenum at the ligament of Treitz attachment
[Ref: Mastery of surgery,6tt’ ed., ch-95, pg. 1089-/,091]

  • Normally, the aortomesenteric angle and aortomesenteric distance are 38-65’and 10-28 mm, respectively.
  • lmaging studies suggest compressionof the third portion of the duodenum as it passesbetween the superior mesenteric artery and the aorta.

Question.4
Class 3 hemorrhagic shock refers to-

AIIMS PG May-2018
A. Class 3 hemorrhagic shock refers to
B. Class 3 hemorrhagic shock refers to
C. Blood loss between 30%-40%
D. Blood loss more than 40%
Correct Ans: C
Explanation

Ans. C. Blood loss between 30%-40%
[Ref: Sabiston textbook ofsurgery,20t’ ed., ch-4, pg. 50]

  Class I Class II Class III Class IV
Blood loss (%) 0-15 15-30 30-40 >40
Central nervous
sysetm
Slightly anx-
ious
Mildly anx-
ious
Anxious or
confused
Confused or
lethargic
Pulse (beats/min) <100 >100 >120 >140
Blood pressure Normal Normal Decreased Decreased
Pulse pressure Normal Decreased Decreased Decreased
Respiratory rate I 4-20/min 20-30/min 30-40/min >35/min
Urine ( niL/hr) >30 20-30 5-15 Negligible
Fluid Crystalloid Crystalloid Crystalloid + lood Crystalloid + lood 

Question.5
An intubated patient with eye opening to pain with abnormal flexion. What is the GCS score?

AIIMS PG May-2018
A. E2Vn1M3
B. E2Vn1M3
C. E2V1M3
D. E2V1M4
Correct Ans: C
Explanation

Ans. C
[Ref Greenbergb Hundbook of neurosurgery, S’t’ ed., ch-18′ pg, 296]

Points’) Best eye pening Best verbal Best motor
6     Obeys
5   Oriented Localizes pain
4 Spontaneous Confused Withdraws to pain
3 To speech Inappropriate Flexion (decorticate)
2 To pain’ Incomprehensible Extensor (decerebrate)
1 None None None’ 

Question.6
The correct procedure of inserting the following equipment in the image given below?

AIIMS PG May-2018
A. Supine with flexed neck
B. Supine with extended neck
C. Sitting with flexed neck
D. None
Correct Ans: C
Explanation

Ans. C. Sitting with flexed neck
[Ref: Shackelfurd’s Surgery of the ALIMENTARY TRACT, th ed., ch-58, pg. 665]

  • For patients who are awake, alert, and cooperative, Fowler position is helpful with a 90- degree angle preferred.
  • A chair may be used, but a stretcher or bed may provide better patient comfort.
  • The patient’s neck should be slightly flexed to avoid endotracheal placement.

Question.7
Where is the second step of damage control resuscitation carried out?

AIIMS PG May-2018
A. In emergency
B. In ICU
C. In OT
D. Prehospital resuscitation
Correct Ans: B
Explanation

Ans. B. In ICU
[Ref: Sabiston furtbook of surgery, 2Ah ed., ch-L6, pg. 417]
Damage Control Sequence

Step 0 Early recognition
  Hemorrhage control
Step 1 Operating Room
  Control Haemorrhage
  Exploration
  Control contamination
  Intra-abdominal packing
  Temporary closure
Step 2 ICU
  Core rewarming
  Correct coagulopaths
  Maximize hemogynamic
  Ventilatory support
  Injury identification
Step 3 Operating Room
  Pack removal
  Definitive repair

Question.8
A 20-year-old male presents with hard painless testicular swellingon investigation AFP is 3080. No paraaortic or iliac nodes as well as no mediastinal lymph nodes found. usG shows uniform echotexture and small areas of necrosis surrounding structures are normal. What is the next best steP:

AIIMS PG May-2018
A. FNAC
B. Trucut bioPsY
C. High inguinal orchidectomy
D. PET-CT
Correct Ans: C
Explanation

Ans. C. High inguinal orchidectomy
[Ref: Sabiston textbook of surgery,2AI’ ed., ch-72, pg. 2102-2104]

  • Initial treatment of suspected testicular tumour is radical inguinal orchiectomy, which involves removal of the testicle and sperrnatic cord at the level of the inguinal ring.
  • Because of the characteristic and well-described lymph drainage of the testicle there is no role or trans-scrotal biopsy or orchiectomy.

Pediatrics

Question.1
A child presents to the emergency with a history of ingestion of button battery, on X-ray it was found in the stomach or duodenum. What is the next step?

AIIMS PG May-2018
A. Endoscopic removal of battery
B. Wait and watch
C. Repeat X-ray after 5 days
D. Immediate laparotomy
Correct Ans: B
Explanation

Ans: B. Wait and watch
[Ref: NBIH Button bsttery ingestion triage and treatment guideline]

  • The management of a Button battery ingestion depends upon the anatomical site of the impacted battery.
  • Button batteries that have cleared the stomach usually pass through the gastrointestinal tract within one week without complications.
  • Follow-up radiographs should be performed in asymptomatic patients who have not passed the battery by 10 to l4 days, regardless ofsize or earlier if patient becomes symptomatic.

Question.2
Absence of which of the following milestone in 3 yr old chitd is called delayed development?

AIIMS PG May-2018
A. Hopping on one leg
B. Hopping on one leg
C. Feeding by spoon
D. Hopping on one leg
Correct Ans: C
Explanation

Ans: C. Feeding by spoon
[Ref: Ghai Essentiul Pediatrics, 8″‘ ed., pg. 49]

  • Hopping on one leg – Should be attained by 4 years ofage
  • Drawing a square – Should be attained by 4Il2 years of age
  • Feeding by spoon Should be attained by l8 months of age
  • Passing a ball to someone- Should be attained by 3 years

Question.3
After the delivery of an infant of diabetic mother, blood glucose of the infant was 60 mg/dt. Which other investigation docs the sister expects that the physician would ask her to do?

AIIMS PG May-2018
A. Serum potassium
B. CBC
C. Serum calcium
D. Serum chloride
Correct Ans: C
Explanation

Ans: C. Serum calcium
[Ref Ghai Essential Pedistrics 8,h ed, pg. 181 und Nelson Textbook of Pediatrics 20h ed” pg. 897]

  • Infants of a diabetic mother are ata higher risk of metabolic complications as compared to normal infants.

These complications include:

  1. Hypoglycemia
  2. Hypocalcemia
  3. Hypomagnesemia
  • Hence the infant needs to be checked for these as soon as possible.

Question.4
An un-immunized 13 months old child comes to you in OPD, according to the latest immunizations schedule, what vaccines will you advise??

AIIMS PG May-2018
A. OPV 3 doses, I IPV 3 Pentavalent and I measles
B. BCG, OPV 3 doses, 3 lPV, 3 Pentavalent and I measles
C. OPV 3 doses, I IPV 3 Pentavalent and 2 measles
D. OPV 3 doses, 3 IPV 3 DPI : Hep-B
Correct Ans: D
Explanation

Ans: D. OPV 3 doses, 3 IPV 3 DPI : Hep-B
[Ref:https://mohfw.gouin/sites/default/files/245453521061489663573.pdf; hup://www.acv ip. o rg/file s/I A P-imm unizution-sc he du le- 2 0 I 6- I P- 2 0 I 6-E p u b. p df]

  • The latest schedule of immunization under NIS is as follows:
  • National Immunization Schedule (NIS) for Infants, children and Presnant women –
    • OPV 3 doses, 3 IPV 3 DPI : Hep-B

Question.5
Correct order ofsuctioning during neonatal resuscitation is?

AIIMS PG May-2018
A. Mouth-Nose
B. Nose-Mouth
C. Mouth-Nose-Trachea
D. Trachea-Nose-Mouth
Correct Ans: A
Explanation

Ans: A. Mouth-Nose
[Ref: Ghai Essential Pediatrics, S’t’ ed., pg. 127]
At the time of birth, if the baby is:

  • Not of term gestatton
  • Not breathing or crying
  • Not having good tone.
  • Then the baby requires resuscitation

Question.6
How long should a child be isolated after being diagnosed with bacterial meningitis to prevent further transmission?

AIIMS PG May-2018
A. Till 24 hours after starting antibiotics
B. Till cultures become negative
C. Till antibiotic course is complete
D. Till l2hrs alter admission
Correct Ans: A
Explanation

Ans: A. Till 24 hours after starting antibiotics
Ref: Ghai Essentisl Pediatrics, 8’t’ ed,, pg. 565′ https://www.cdc.gov
Prevention of transmission:

  • Droplet precautions for the first 24 hours of antimicrobial therapy is sufficient

Question.7
You have been called to declare a brain dead l2-year-ord chitd in pICU, all ofthe given are signs of brain death except?

AIIMS PG May-2018
A. Normal BP without pharmacological support
B. Positive spinal reflexes on stimulation
C. Sweating and tachycardia
D. Decorticate and decerebrate posturing
Correct Ans: D
Explanation

Ans: D. Decorticate and decerebrate posturing
[Ref Dhanwate AD. Brainstem death: A comprehensive review in Indian perspective’ [ndian Journal o/ Critical Care Medicine : Peer-reviev,ed, o.fficiat pul.tlicalion of Indian Society of Critical Care Medicine. 2014;18(9):596-605. doi:10.4103/0972-5229.140151. Goila AK, Pawar M. The diagnosis of brain death. Indian Jrurnal of Critical Care Medicine; Peer-reviewed, o.fficial publication of Indian Society of Critical Corc Madicine. 2009;I 3(I ):7-11. doi: I 0.4103/0972-5229.53 I0t]
Diagnosis of brain death in India:
Who should diagnose:

  • Team of four medical experts including
  • Medical Administrator In charge of the hospital.
  • AuthorizedSpecialist
  • Authorized Neurologist/Neuro-Surgeon
  • Medical Officer treating the patient.

Question.8
A 2-year-old child with history of fall one year back with parietal bone fracture now presented with painful and growing parietal swelling.

AIIMS PG May-2018
A. Growing scalp hematoma
B. Growing fracture
C. Subdural hygroma
D. Chronic abscess
Correct Ans: B
Explanation

Ans: B. Growing fracture
[Ref: Growing skull fractures: classiJication and management. Naim-UFRahman etsl. Br J Neurosurg. (1994)]
Growing Fractures:

  • Also known as traumatic encephaloceles or leptomeningeal cysts
  • Skull fractures associated with an underlying dural tear may fail to heal properly.

Question.9
Craniopagus is defined as fusion of:

AIIMS PG May-2018
A. Head and spine
B. Head only
C. Thorax and spine
D. Thorax only
Correct Ans: B
Explanation

Ans: B. Head only
[Ref: Coran Pediatric Surgery 7u ed., pg. 1728]
Craniopagus:

  • Extent of union:Cranial neuropore
  • Skull venous sinus &meninges l00%
  • Cerebral cortex 37%

Question.10
An adolescent school girl complaints of dropping objects from hands, it gets precipitated during morning and during exams. There is no history of loss of consciousness and her cousin sister has been diagnosed with epilepsy. EEG was done and was suggestive of epileptic spikes. What is the diagnosis?

AIIMS PG May-2018
A. Juvenile myoclonic epilepsy
B. Atypical absence
C. Choreo-athetoid epilepsy
D. Centrotemporal spikes
Correct Ans: A
Explanation

Ans: A. Juvenile myoclonic epilepsy
[Ref: Nelson Textbook of Pediatrics, 20′ ed., pg. 2836]
Juvenile myoclonic epilepsy (Janz syndrome):

  • Starts in early adolescence with 1 or more of the following manifestations:
  • Myoclonic jerks in the morning, often causing the patient to drop things
  • Generalized tonic–clonic or clonic–tonic–clonic seizures upon awakening; and Juvenile absences.
  • Sleep deprivation, alcohol (in older patients), and photic stimulation, or, rarely, certain cognitive activities (such as exams) can act as precipitants.

Obs / Gyne

Question.1
A female is on Mala N for contraception. After the end of the first strip, there was no withdrawal bleeding. What should be done?

AIIMS PG May-2018
A. Start next cycle oftablets frorn 5th day
B. Start next cycle from next daY
C. Urgent visit to hospital and check for pregnancy
D. Take two pills afier 12 hours
Correct Ans: C
Explanation

Ans. C. Urgent visit to hospital and check for pregnancy
[Ref IIHO guidelines; CDC guidelines]
After ruling out pregnancy, next cycle of OCPs can be started.


Question.2

A pregnant female with known cardiac disease presents to you in the first trimester with history of warfarin embryopathy what should be advised now?

AIIMS PG May-2018
A. Continue warfarin throughout the pregnancy
B. Replace warfarin with heparin in First trimester
C. Give acicoumarin
D. Use LMW heparin
Correct Ans: B
Explanation

Ans. B. Replace warfarin with heparin in First trimester
[Ref: Williams Obstetrics, 24’h ed]

  • Warf’arin has a low molecular weight and readily crosses the placenta.
  • Exposure between the 6th and 9th weeks may result in warfarin embryopathy characterized by stippling of the vertebrae and femoral epiphyses and by nasal hypoplasia with depression ofthe nasal bone.

Question.3
Kartagener syndrome all true except?

AIIMS PG May-2018
A. Bronchitis
B. Sinusitis
C. Bronchiectasis
D. Infertility
Correct Ans: A
Explanation

Ans. is ‘a’ i.e., Bronchitis

  • Kartagener syndrome is a subset of primary ciliary dyskinesia, an autosomal recessive condition characterized by abnormal ciliary structure and/or function leading impaired mucociliary clearance.
  • Kartagener is a primary ciliary disorder and as such the sperms have abnormal/reduced motility (Asthenozoospermia).
  • Blockage of epididymis is a feature of Young syndrome (But there is no Situs inversus like Kartagener syndrome and patients tends to have normal sperm motility)

Clinical presentation

  • Kartagener syndrome is characterised by the clinical triad of 1
  • Situs inversus
  • Chronic sinusitis and/or nasal polyposis
  • Bronchiectasis

Other features include

  • Telecanthus: widened interpupillary distance by a nasal polyp
  • Infertility in males
  • Subfertility in females

Question.4
A 25-year-old lady with submucosal fibroid was undergoing myomectomy. The surgeon was using 1.5 yo glycine as irrigating fluid for the cavity. During the surgery the nurse informs the surgeon that there is a 500m1 fluid deficit. What is the next step to be done?

AIIMS PG May-2018
A. Stop the’surgery
B. Change the fluid to normal saline
C. Continue the surgery with careful monitoring of fluid status
D. Give furosemide to the patient and continue surgery
Correct Ans: C
Explanation

Ans. C. Continue the surgery with careful monitoring of fluid status
[Ref. BSGE/ESGE guideline on manage,nent of fluid disrension metlia in operative hysteroscopy]

  • Asymptomatic hypervolemia can be managed by fluid restriction with or without diuretics.
  • Patient should be observed for symptoms of hyponatremia and continued electrolyte monitoring should be done.

Question.5
While performing Burch operation there was significant bleeding and pooting of blood in the space of Retzius. The source of bleeding cannot be visualized. What is the next step in the management?

AIIMS PG May-2018
A. Call vascular stlrgeon
B. Give a generalized suture in bleeding area
C. Lift endopelvic fascia by putting fingers in vagina
D. Placing surgical drain
Correct Ans: C
Explanation

Ans. C. Lift endopelvic fascia by putting fingers in vagina

  • Burch colposuspension (retropubic urethropexy) involves the attachment of the fascia at the level of the bladder neck to the iliopectineal ligament (Cooper’s ligantent).
  • lt is an abdorninally perfbrmed surgery for stress urinary incontinence .

Question.6
Which of the following is the important marker of male infertility in semen analysis?

AIIMS PG May-2018
A. Motility
B. Concentration
C. Volume
D. Sperm Count
Correct Ans: B
Explanation

Ans. B. Concentration
[Ref: Clinical Gynecologic Endocrinology Infertility, 8′ ed]
Sperm concentrationand progressive motility is important in distinguishing fertile from infertile men but strict sperm morphology is one of the most discriminating value.


Question.7

A 32 weeks pregnant female presented with labor pains and minimal vaginal discharge, on analysis of the cervicovaginal discharge showed fibronectin. What is the probable diagnosis?

AIIMS PG May-2018
A. Preterm labour
B. IUGR
C. IUD
D. Cervical infection
Correct Ans: A
Explanation

Ans. A. Preterm labour
[Ref: Williams Obstetrics 24′ ed]

  • Fibronectin can be detected in cervicovaginal secretions before membrane rupture and is a marker for impending preterm labour.
  • It reflects stromal remodelling of the cervix before labour.
  • It is measured using an enzyme-linked immunosorbent assay, and values exceeding 50 ng/mL are considered positive

Question.8
When would you do trans-vaginal sonography in post-menopausal bleeding if endometrial thickness is?

AIIMS PG May-2018
A. 5mm
B. 7mm
C. 4mm
D. 9mm
Correct Ans: A
Explanation

Ans. A. 5mm

[Ref: RCOG guidelines 2012]


Question.9

A 76-year-old female presented with non-healing ulcer on labia majora for 6 months measuring 2×3 cm with no palpable lymphadenopathy. Biopsy shows squamous cell carcinoma. Management includes?

AIIMS PG May-2018
A. Radical vulvectomy with unilateral LN dissection
B. Radical vulvectomy with bilateral LN dissection
C. Simple vulvectomy
D. Chemoradiation with resection
Correct Ans: B
Explanation

Ans. B. Radical vulvectomy with bilateral LN dissection
[Ref: Bereks and Novaks gtnuecologt, I5’t’ ed., RCOG guidelines FIGO staging 2009]
Locally advanced vulvar cancer (bulky stage III and stage IV)

  • Radical surgery (radical vulvectomy plus bilateral lymphadenectomy):
  • If partial removal of other involved structures is needed (e.g., urethra, vagina, anus, bladder, rectum) and/or pelvic exenteration is necessary, consider preoperative chemoradiation

Chemoradiation (with or without subsequent completion surgery):

  • This approach has been shown to decrease the need for exenterative surgery.

Question.10
Absolute contraindication for CUT insertion?

AIIMS PG May-2018
A. Unmotivated person
B. Menorrhagia
C. Previous ectopic
D. Previous history ofabortion
Correct Ans: A
Explanation

Ans. A. Unmotivated person
[Ref: ACOG guideline 187, November 2017]

  • CuT is MEC I in ectopic pregnancy, previous history of abortion so it is safe.
  • Unmotivated person is absolute contraindication for any procedure.

Question.11
A 14 year girl presented with absent thelarche.On examination uterus was present. Investigations showed high FSH. karyotype is XY.What is the probable diagnosis?

AIIMS PG May-2018
A. Gonadal dysgenesis
B. Kallman syndrome
C. Androgen insensitivity syndrome
D. Adrenal hyperplasia
Correct Ans: A
Explanation

Ans. A. Gonadal dysgenesis
[Ref: Clinical Gynecologic Endocrinoktgt Infertility, 8″ ed]
Gonadal dysgenesis (Swyer Syndrome):

  • Uncommon form of gonadal dysgenesis, characterized by a 46,XY karyotype.

Question.12
A 2s-year-old women underwent induced ovulation.on uSG, ovary showed 8 follicles. Serum estradol level was 800 pg/ml.what is the next step in the management of this patient?

AIIMS PG May-2018
A. Retrieve follicles
B. Give cabergoline
C. cancel cycle
D. withhold HCG
Correct Ans: A
Explanation

Ans: A. Retrieve follicles
[Ref: Berek and Novak’s Gynaecologt]

  • Day of analysis of above measurements and size of fbllictes are not specified in the question. So, presuming these parameters to be treasured on the mid cycle, next step according to the options would be to retrieve follicles.

Question.13
A pregnant woman with G3P2L0 presented to you with a pregnancy at period of gestation of 9 weeks. She has a history of conization one year back currently on follow up with no recurrence on PAP smean She also has the history of preterm births at 30 and 32 weeks during her last 2 pregnancy. What is your next step in the management of this patient?

AIIMS PG May-2018
A. USG to see cervical length
B. Cervical cerclage
C. Complete bed rest
D. Abdominal cerclage
Correct Ans: A
Explanation

Ans: A. USG to see cervical length
[Ref NICE guidelines; ACOG guidelines; SOGC guidelines]
The current review fromACOG recommends cervical cerclage for women with a current singleton pregnancy, prior spontaneous preterm birth at less than 34 weeks of gestation, and cervical length less than 25 mm.


Question.14

A pregnant female presents with prolonged labor in emergency.She is taken for cesarean section. What is the correct position in which the nurse should keep the patient on OT table?

AIIMS PG May-2018
A. Supine with wedge under right hip
B. Semi fowlers
C. Trendelenburg with legs in stirrup
D. Prone position
Correct Ans: B
Explanation

Ans: B. Semi fowlers
[Ref DC Dutta’s textbook of Obstetrics, gtt’ ed]
In caesarean section, patient is placed in supine position. In order to prevent venocaval compression, l5-degree tilt is given by placing a wedge under the right hip till delivery of the baby


Question.15

A female come to gynaeoPD for preconceptual counseling, with history of two second trimester abortions. What is the next investigation you will advice

AIIMS PG May-2018
A. TVS
B. hysteroscopy
C. Endometrial biopsy
D. chromosomal abnormalities
Correct Ans: A
Explanation

Ans: A. TVS
[Ref: Williams Obstetrics 24h ed”]

  • Most common cause of second trimester abortion is cervicouterine abnormalities.
  • Next step would be to do an ultrasound and look for any structural uterine anomaly.
  • Chromosomal abnormalities are common cause of aboftions in first trimester.

Question.16
A pregnant female delivered a baby with normal expulsion of an intact placenta. After half hour she started bleeding per vaginaly. On examination she was hypotensive and boggy mass is palpated per abdomen. USG showed retained placental tissues. what is the likely diagnosis?

AIIMS PG May-2018
A. Placenta succenturiata
B. Adenomyosis
C. Placenta accreta
D. Membranous placenta
Correct Ans: A
Explanation

Ans: A. Placenta succenturiata
[Ref: DC Dutta’s turtbook of Obstetrics, gh ed]

  • Placenta succenturiata has one (usual) or more small lobes of placenta placed at a varying margin from the main placental margin.
  • A leash of vessels connects the small lobe with the main lobe.
  • Many times, succenturiate is retained and it presents as postpartum hemorrhage which may be primary or secondary.

Question.17
Foltowing are the features of the color of normal amniotic fluid during delivery?

AIIMS PG May-2018
A. Milky to yellowish green with mucus flakes
B. Amber colored
C. Clear colorless to Pale Yellow
D. Golden color
Correct Ans: C
Explanation

Ans: C. Clear colorless to Pale Yellow

[Ref: DC Dutto’s textbook of Obstetrics, 9’t’ ed]

  • Green yellow with flakes (meconium stained)- Fetal distress
  • Golden color- Rh incompatibilityGreenish Ye11ow (saffron)- postmaturity
  • Dark colored – concealed accidental hemorrhage
  • Dark brown (tobacco juice)- Intruterine demise

Question.18
A lady presented with 7 weeks amenorrhea presented with slight vaginal spotting. CRL was 5mm with well-formed gestational sac with calculated GA of 5.6 weeks on TVS. Next line of management?

AIIMS PG May-2018
A. Wait for another I week and repeat TVS
B. Surgical or medical evacuation
C. Wait for another 4 weeks
D. Serum hCG levels
Correct Ans: A
Explanation

Ans: A. Wait for another I week and repeat TVS
[Ref: Williams ohstetrics, 24’h ed]

  • An intrauterine gestational sac is reliably visualized with transvaginal sonography by 5 weeks, and an embryo with cardiac activity by 6 weeks.
  • The embryo should be visible transvaginally once the mean sac diameter has reached 20 mm, otherwise the gestation is anembryonic.
  • Cardiac motion is usually visible with transvaginal imaging when the embryo length has reached 5 mm. If an embryo less than 7 mm is not identified to have cardiac activity, a subsequent examination is recommended in 1 week.

Question.19
All of the following indicate Fetal lung maturity except?

AIIMS PG May-2018
A. LecithiniSphingomyelin ratio >2
B. Positive shake test
C. Increased phosphatidyl glycerol
D. Blue cells in Nile Blue Test
Correct Ans: D
Explanation

Ans. D. Blue cells in Nile Blue Test
[Ref: DC Dutla’s textbook of Obstetrics,9’t’ ed]
All of the above are correct except that it is the presence of more than 50 % orange coloured cells in Nile Blue test that suggests fetal pulmonary maturity.

Orthopedics

Question.1

A 55-year-old male presents with severe backache for 10 days and urinary incontinence with a H/o Intervertebral lumbar disc prolapse. There is no H/o fever or weight loss. What is the likely diagnosis?

AIIMS PG May-2018
A. Potts spine
B. Multiple myeloma
C. Cauda equine syndrome
D. Bone metastasis
Correct Ans: C
Explanation

Ans: C: Cauda equine syndrome
[Ref: Apley!c system of orthopaedics and fracture 9″ ed., pg. 246,480]
Cauda equina is tuft of fibres which begins at the end of spinal cord.
Compression over this part may cause cauda equina syndrome.
Causes of Cauda equina syndrome are:

  • Lumbar disc herniation, Spinal canal stenosis, Trauma, Abscess etc.

Question.2
A 55 year old female came with hip flexor contracture. What is the most likely test to be done in this case?

AIIMS PG May-2018
A. Allis test
B. Thomas test
C. Ober test
D. Trendlenberg test
Correct Ans: B
Explanation

Ans : B : Thomas test
[Ref. Apley’s system of orthopaedics and fracture 9’h ed., pg. 495]

  • Full name of test is Hugh owen well leg hip flexion Thomas test. This test is to find out fixed flexion deformity of hip joint.
  • Other test to determine Iliotibial band contracture.
  • Trendelenberg test is to find out abductor weakness of hip joint
  • Allis test or Galleazi sign is used to diagnose DDH in children

Question.3
A Patient of supracondylar humerus fracture is unable to flex interphalangeal joint of the thumb. Which nerve is most likely injured?

AIIMS PG May-2018
A. Median nerve
B. Superficial branch of ulnar nerve
C. AIN
D. PIN
Correct Ans: C
Explanation

Ans: C: AIN
[Ref: Apley’s system of orthopaedics and fracture 9’h ed., pg. 750,758-760]

  • Flexion at IP joint of thumb is caused by Flexor Pollicis Longus which is supplied by Anterior Interooseous Nerve a pure motor branch of Median nerve.
  • Also AIN is most common nerve to be injured in supracondylar fracture of humerus while ulnar nerve is least common.
  • But in Flexion type of injury to supracondylar fracture most common nerve to be damaged is ulnar nerve.

Image Based Question

Question.1
Which of the following is the marked structure in the below picture?

AIIMS PG May-2018
A. Neurotransmitter
B. Synaptic vesicles
C. Microtubule
D. Collagen fibril
Correct Ans: B
Explanation

Ans. b. Synaptic vesicles

[Ref: Z. Taoufiq, OIST 2013]

  • An ultramicroscopic image shows an isolated synapse from a brain sample before mass spectrometry analyses.
  • The pre-synapse typically shows many vesicles containing neurotransmitters kept attached to the post-synapse.

Question.2
Which of the following projects efferent fibers through the marked structure?

AIIMS PG May-2018
A. Hippocampus
B. Mammillary body
C. Caudate nucleus
D. Amygdala
Correct Ans: A
Explanation

Ans. A. Hippocampus
[Ref: Grays Anatomy, 41″‘ ed, pg. 439. Snell’s neuroanator?E 7h ed, pg. 310. Barr’s neuroanatomy, IAh ed”, pg. 273]

  • The marked structure is fomix.
  • The human fornix contains more than I million myelinated axons.
  • Most of these axons originate in the subiculum.
  • The rest of the axons originate in the hippocampus or are afferent to the hippocampal formation.

Question.3
Identify the ions based on conductance graph of a neuron shown below?

AIIMS PG May-2018
A. Na, k
B. K, Na
C. Na, Ca
D. Ca,Na
Correct Ans: A
Explanation

Ans.A. Na, k
[Ref: Ganong’s Review of Medical Physiologt,25h ed., ch-4, pg.9I]

  • Na channels are fast activating and are responsible for depolarisation phase of action potential
  • K channels are responsible for repolarisation phase of action potential.


Question.4

Choose the correct statement regarding the image given below

AIIMS PG May-2018
A. B has done more work than A
B. B has both preload and after load added to it
C. B has undergone maximum tension
D. The maximum tension generated will be in A
Correct Ans: A
Explanation

Ans. A. B has done more work than A

  • A: Preload
  • B: Afterload.
  • In preloaded condition, the relaxed state.
  • In afterloaded condition, the load acts on the muscle only when its actively contraction and is supported during the relaxed state.
  • External work done: Load x displacement
  • In this diagram displacement is higher in B so more work is done there.

Question.5
During exercise in physiological limits what is the effect on end systolic volume?

AIIMS PG May-2018
A. ESV decreases
B. ESV increases
C. ESV remain unchanged
D. ESV first decreases and then increases
Correct Ans: A
Explanation

Ans. A. ESV decreases

  • During exercise there is an increase in preload (end diastolic volume)- increased venous return.
  • Increase in afterload – increased Mean BP and increased stroke volume due to improved myocardial contractility.
  • Theretbre, the ESV is decreased in exercise

Question.6
A middle aged immune compromised male came with fever and breathlessness. HRCT showed a middle lobe lesion with infiltration. Lung biopsy from the lesion is shown in image. Most likely Diagnosis is?

AIIMS PG May-2018
A. Tuberculosis
B. Cryptogenic organizing pneumonia
C. Small cell carcinoma luns
D. CMV pneumonia
Correct Ans: D
Explanation

Ans. D. CMV pneumonia
[Ref. Robhins & Cotran,9’L ed., pg. 687, 705 & 717]

  • The history of immune compromise is provided in the history.
  • In the given image, many cells with intranuclear inclusions, suggestive of cytomegalovirus are seen.

Question.7
A worker was working in a factory from the past 20 years t and now presented with pleural thickening and fibrosis. Histopathology of the lesion is shown in below image. Most likely diagnosis is ?

AIIMS PG May-2018
A. Asbestosis
B. Cotton Fiber
C. Coal Worker Pneumoconiosis
D. Silicosis
Correct Ans: A
Explanation

Ans. A. Asbestosis
[Ref: Robbins & Cotran,9t’ ed., pg. 691,692]

  • In the given microphotograph, there is diffuse interstitial fibrosis with presence of asbestos body.
  • In asbestosis, after phagocytosis by macrophages asbestos fibers activate the inflammasome and stimulate the release of proinflammatory factors and fi brogenic mediators.
  • Diffuse pulmonary interstitial fibrosis, indistinguishable from diffuse interstitial fibrosis resulting from other causes, except for asbestos bodies which are golden brown, fusiform or beaded rods with a translucent center consisting of asbestos fibers coated with an iron- containing proteinaceous material.

Question.8
Toluidine blue staining is used for identification of ?

AIIMS PG May-2018
A. Mast cell
B. Melanocyte
C. Fibroblast
D. Macrophages
Correct Ans: A
Explanation

Ans. A. Mast cell
[Ref: Quick Reference Handbookfor Sargical Pathologists, 2011, pg. 75]

  • Toluidine blue stains the Mast cells. Their cytoplasm contains metachromatic granules composed of heparin and histamine.
  • It stains the mast cells violet with blue background.

Question.9
Which of the following labels corresponds to the condenser of the microscope?

AIIMS PG May-2018
A. C
B. B
C. D
D. A
Correct Ans: A
Explanation

Ans. A. C

  • In the given image, C corresponds to condenser of microscope.
  • Different parts of microscopes are marked in the following image.


Question.10

A 12-year-old boy had a cut in his forearm 4 days ago. Now the bleeding has been stopped due to granulation tissue formation. While taking a skin biopsy a part of the granulation fissue was also included in the specimen. The histology of granulation tissue is shown below. Which type of collagen is found in this granulation tissue?

AIIMS PG May-2018
A. Type I
B. Type 2
C. Type 3
D. Type 4
Correct Ans: C
Explanation

Ans. C. Type 3
[Ref: Robbins & Cotran,9h ed., pg. 23 & I0B]

  • Granulation tissue fill in the gaps ofwound and provide the underlyingframework for the regrowth of tissue epithelium.
  • During wound healing, first a provisional matrix containing fibrin, plasmafibronectin, and type III collagen is formed, but in about2 weeks this is replaced by a matrix composed primarily of type I collagen.

Question.11
Identify the organism from the given life cycle?

AIIMS PG May-2018
A. Influenza Virus
B. RSV
C. Para-influenza
D. Sars virus
Correct Ans: A
Explanation

Ans. A. Influenza Virus

Influenza virus life cycle:


Question.12
Identify the organism on Bacitracin sensitivity disk test given below?

AIIMS PG May-2018
A. Streptococcus pyogenes
B. Staph aurus
C. Clostridium
D. Corynebacterium
Correct Ans: A
Explanation

Ans. A. Streptococcus pyogenes

  • Bacitracin test is used to determine the effect of a small amount of bacitracin (0.04 IU or 0.05 IU not higher) on an organism.
  • Streptococcus pyogenes (Group A Streptococci/ is inhibited by the small amount of bacitracin in the disk; other beta-hemolytic streptococci usually are not.

Question.13
A known HIV patient on anti-retroviral therapy presented with diarrhea of six months duration. Stool microscopy was done in which 10-30 micrometer cysts were seen, Kinyoun stain was positive. What is the most likely diagnosis?

AIIMS PG May-2018
A. Cystoisospora
B. Cryptosporidium
C. Balantidium coli
D. Strongyloides
Correct Ans: A
Explanation

Ans. A. Cystoisospora

  • Comparison of Coccidian Parasites Causing Diarrhea in Immunocompromised Host.
Property Crytosporidium
Infective form
  • Sporulated oocyst Thick walled  oocyst (80%) by contaminated food and water.
  • Thin walled (20%) Thin walled (20%)
 Sporulated oocyst  4-6 p.m, round contains four sporo-zoites
 Acid fastness Detection limit > 50,000 oocyst/ml stool  Uniformly acid fast
 Property Crytosporidium
Autofluorescence  No, but can be stained with fluores cent dye
 Sporulation of the oocyst  Occurs inside the host cells (entero- cytes)
 Diagnostic form  Sporulated oocyst
 Sporulated oocyst  Seen
 Treatment Nitazoxanide
 Outbreaks  Common

Question.14
A child with l0 days atldominal pain presented to OPD. Stool microscopy was done which showed the given findings. What is the DOC for the disease caused by the given organism?

AIIMS PG May-2018
A. Albendazole
B. Mebandazole
C. Praziquintal
D. Pyrantelpamoate
Correct Ans: C
Explanation

Ans. C. Praziquintal
Treatment of Cestodes:

  • Praziquantel is the DOC of all cestodes followed by Niclosamide. Except, Hydatid disease and neurocysticercosis: Albendazole.

Question.15
A patient was suffering from hepatorenal syndrome. A urine sample was obtained from the patient and was examined under the microscope. Which of the following technique is treing used to make the organism visible?

AIIMS PG May-2018
A. Dark field microscopy
B. Routine microscopy with negatively stained background
C. Phase contrast microscopy
D. Compound light microscopy
Correct Ans: A
Explanation

Ans. A. Dark field microscopy

  • Leptospira are extremely thin, hence seen under dark ground microscope.
  • They are tightly and regularly coiled, with characteristic hooked ends like umbrella handle.
  • They are highly motile; exhibit spinning and translational movements.

Question.16
Identify the organism which shows effacement and attachment of intestinal cells as shown below?

AIIMS PG May-2018
A. EPEC
B. ETEC
C. Enterohemorrhagic ecoli
D. Diffusely enteroadherent ecoli
Correct Ans: A
Explanation

Ans. A. EPEC
Enteropathogenic E.coli (EPEC)

  • EPEC frequently cause infantile diarrhea.
  • Nontoxigenic and noninvasive

Mechanism of diarrhea:

  • Adhesion to intestinal mucosa mediated by plasmid coded bundle-forming pili.
  • A/E lesions (attaching & effacing lesions) on the intestinal epithelium.

Question.17
A patient presented with complaints of dysphagia. On UGI Endoscopy a diagnosis of esophageal candidiasis was made. The organism obtained from the sample was cultured on a specialized media and the below findings were visible on microscopy of the culture. What phenomenon is shown below?

AIIMS PG May-2018
A. Germ tube formation
B. Spore formation
C. Mould to yeast formation
D. Budding
Correct Ans: A
Explanation

Ans. A. Germ tube formation
Tests for species identification:
Germ tube test:

  • Reynolds Braude phenomenon, specific test for C. albicans.
  • It is differentiated from pseudohyphae as there is no constriction at the origin.
  • Though the test is specific for c.albicans. it may also be positive for c. dubliniensis.

Question.18
Choose the icorrect statement regarding the given image?

AIIMS PG May-2018
A. Causes AV block,
B. Atropine is antidote
C. Only root is poisonous
D. First symptom is sweet taste
Correct Ans: D
Explanation

Ans: D. First symptom is sweet taste

  • Aconite roots are usually mistaken for horseradish root., Active principles: Aconitine0, pseudo-aconitine, indaconitine, picraconitine and aconine.No odour, sweet taste (mitha bish)
  • The initial signs are gastrointestinal, including nausea, vomiting, and diarrhea. This is followed by a sensation of burning, tingling, and numbness in the mouth and face, and of burning in the abdomen.[3] In severe poisonings, pronounced motor weakness occurs and cutaneous sensations of tingling and numbness spread to the limbs 
  • Given flower belongs to the plant Aconite which is a cardiac poison.
  • Aconitine binds with the voltage-dependent sodium-ion channels.
  • Aconitine first stimulates and then paralyze the peripheral terminations of sensory, secretory and motor nerves.
  • Tingling followed by numbness of mouth and throata is characteristics ymptom of aconite poisoning

Question.19
  Match the following columns 
A- (1) Adipocere
B-(2) liver mortis
C- (3) mummification
D-(4) marbling
 
 

AIIMS PG May-2018
A. A-1 B-2C-3 D-4
B. A-4 B-l C-2D-3
C. A-4 B-2 C-l D-3
D. A-2 B-4 C-3 D-l
Correct Ans: D
Explanation

Ans: D. A-4 B-3 C-2 D-l
Correct matching is below: 
Livor Mortis – Postmortem Staining:

  • Marbling (Linear Branching Patterns on the Skino)

 

  • Adipocere (Saponification):

  • Mummification (Drying and Dehydration of the Body):

 


Question.20
A 14 year old female was claimed to be kidnapped, on interrogation she claims that she is not a minor and left home with the person on her own will. Court ordered for her age estimation. Given below are the X-rays of pelvis, wrist and bilateral elbow. What is her most probable age based on these?


AIIMS PG May-2018
A. 14 years
B. l6-17 years
C. l7-19 years
D. 2l-22 years
Correct Ans: D
Explanation

Ans. D. 21-22 years

  • In figure 1, all the ossification centres are fused around elbow, so the age must be> 16 years.
  • In figure 2, lower end of radius and ulna are fused, so the age must be atleast 18 years.
  • In figure 3, Ischial tuberosity isfused, so the age must be around 20-22 years.
  • So the best answer for this question would be option (d) 21-22 years

Question.21
Order of least Margin of error in the graph given below is ?

AIIMS PG May-2018
A. 3>2>l
B. 3>t>2
C. l>3>2
D. l:2:3
Correct Ans: A
Explanation

Ans: A. 3>2>l
[Ref Biostatistics Principles and Prsctice by B. Antonisamy and Mc graw hill publication, pg. 44-45]

  • Given image shows three normal distribution curve of different sample size.
  • All three curves have same mean but different standard deviation.
    • As the sample size is decreasing, flatness of curve increasing and peak of curve diminishing which signifies wider confidence interval for given mean.
  • Narrower the base of curve, lesser the margin of error will be.
  • ln given question, curve I has least margin of error followed by curve 2 and curve 3 shows highest margin of error.
    • Hence correct answer is 3>2>1.

Question.22
Identify the structure marked as A?

AIIMS PG May-2018
A. Incus
B. Stapes footplate
C. Lateral semi circular canal
D. Malleus
Correct Ans: D
Explanation

Ans: D. Malleus
The other structures in the image shown are

  • Incus
  • Stapes

Question.23
A patient is taking drugs for rheumatoid arthritis and has a history of cataract surgery lyear back, the patient presented with sudden painless loss of vision, probable diagnosis is?

AIIMS PG May-2018
A. CME
B. Chloroquine toxicity
C. Macularhole
D. Chronic choroiditis
Correct Ans: A
Explanation

Ans: A. CME
[Ref 2017 Jul 28, medicine, Baltimore journal of ophthalmology]

  • Most probably leakage around macula shown in given image is most likely due to CME.
  • Macular hole is different entity having no association with rheumatoid arthritis & have different fundal picture.
  • Choroiditis shows healed scars or oatches in retina which have more likely infectious causes.

Question.24
A woman presents with pain, swelling and redness of knee joint and hand. There were associated complaints of Morning stiffness with problem in squatting. The swelling spared the DIP joints. The image of the patient’s hands is shown below. What is the most likely diagnosis?

AIIMS PG May-2018
A. Rheumatoid Arthritis
B. Heberden nodes on preexisting osteo arthritis
C. Tenosynovitis of extensor tendon of MCP joints
D. Post traumatic dystrophy with Complex regional pain syndrome
Correct Ans: A
Explanation

Ans: A. Rheumatoid Arthritis
[Ref: Harrison, 18tL Ed., pg. 2738-274 1 , 2739f, 2740]
Common Joint Deformities in RA
Boutonniere deformity:

  • The middle finger joint bends toward the palm while the outer finger joint may bend opposite the palm.

Swan-neck deformity:

  • The base of the finger and the outermost joint bend, while the middle joint straightens.

Hitchhiker’s thumb:

  • The thumb flexes at the metacarpophalangeal joint and hyperextends at the interphalangeal joint below your thumb nail.
  • It is also called Z-shaped deformity of the thumb.

OA most commonly affects three parts of the hand:

  • The trapeziometacarpal [TMC] or carpometacarpal [CMC] joint
  • The distal interphalangeal [DIP]joint
  • The proximal interphalangeal [PIP]joint

Question.25
What is the diagnosis based on the ECG given below?

 
 

AIIMS PG May-2018
A. AF
B. PSVT
C. Multifocal atrial tachycardia
D. Ventricular Fibrillation
Correct Ans: A
Explanation

Ans: A. AF
[Ref Harrison, 18’t’ ed., pg. 1878-1879, 1878f]

  • RR intervals are not regular.
  • Also, P wave is not appreciable.
  • So, it’s the ECG of atrial fibrillation with fast ventricular rate.

Question.26
A patient presented with fever for 8 days and difficulty in breathing for two days, on examination there is red painless rash found on the left side of the chest. Treatment will be –

AIIMS PG May-2018
A. Oseltamivir
B. Streptomycin
C. Doxycycline
D. Ceftriaxone
Correct Ans: C
Explanation

Ans: C. Doxycycline
[Ref: Harrison Iff ed”, pg. 1408t, I4I3]

  • The presence of rash over the chest is typically an eschar, which is black necrotic, painless.
  • Usually it is present in axilla, groin region.
  • Rickettsial infections are caused by various bacterial species from the genera Rickettsia, Orientia, Ehrlichia, Neorickettsia, Neoehrlichia, and Anaplasma.
  • The drug most commonly used is doxycycline or tetracycline, but Azithromycin or chloramphenicol is an altemative.

Question.27
A patient of RTA presents in emergency with inability to speak. On examination, the patient was moaning with inability to speak but patient was able to understand what he wanted to speak. Which of following marked area of brain is involved in this?

AIIMS PG May-2018
A. A
B. B
C. C
D. D
Correct Ans: B
Explanation

Ans: B. B
[Ref: Harrison, I&h ed., pg. j236]
Patient comprehension is intact in this case, but he has difficulty in speech, resulting in development of  Broca’s aphasia.


Question.28

The given image shows methylene blue being injected in the peritumoral region. Which of the given procedure is being performed?

AIIMS PG May-2018
A. Sentinel lymph node biopsy
B. Tumour painting
C. Breast tattooing
D. Peritumour marking with dye
Correct Ans: A
Explanation

Ans. A. Sentinel lymph node biopsy
[Ref: Master of surgery,6’t’ ed., ch-49, pg. 591-594]

  • Sentinel node biopsy is based on the concept that an afferent lymphatic from a primary tumour drains first to the “‘sentinel” node befbre reaching second-tier (non-sentinel) nodes in the regional nodal basin.
  • Because this sentinel node is the first node encountered by tumour cells as they metastasize to the regional nodal basin.
  • It represents the tumour status of the entire nodal basin. Selective identification removal.
  • And histopathologic analysis of sentinel nodes therefore can be used to identify occult nodal metastasis without undertaking complete lymph node dissection.

Question.29
A 50 years old male presented in the emergency with high BP (160/100) and heart rate of 120. A CECT is done which is given below. what is the management of the given condition?

AIIMS PG May-2018
A. Surgical repair
B. Beta blocker medication
C. Low molecular weight heParin
D. Vitamin k inhibitors
Correct Ans: A
Explanation

Ans. A. Surgical repair
[Ref: sabiston textbook ofsurgery 2Ah edition, ch-61, pg. 1746-1747 and Kaufman J & Lee M Vascular & Intementional Radiologt: The Requisites, Mosby 2004]

  • The given CT Scan shows Type A Aortic Dissection.
  • Type A dissection may also result in coronary artery occlusion, aortic regurgitation and pericardial tamponade and therefore management of this type of dissection is usually emergency surgical repair.
  • Type B dissections are usually managed with aggressive brood pressure control unless there are complications.

Question.30
A 28-week Pregnant female presents with the fetal distress on .examination and the test performed (MCA Doppler study) is given below. What should be the next step in management?

AIIMS PG May-2018
A. Immediate termination of pregnancy
B. Give steroid cover and monitor with doppler and BPP, and plan delivery
C. Give steroid and take up for CS immediately
D. Go for normal vaginal delivery as baby is very small
Correct Ans: B
Explanation

Ans. B. Give steroid cover and monitor with doppler and BPP, and plan delivery

  • Doppler shows absent blood flow during diastole in the brain.
  • In the normal situation, the fetal MCA has a high resistance flow which means there is minimal antegrade flow in foetal diastole.
  • In pathological states this can turn into a low resistance flow mainlv as a result ofthe fetal head sparing theory

Question.31
Identify the instrument shown?

AIIMS PG May-2018
A. Episiotomy scissor
B. Babcock forceps
C. Cutting scissors
D. Ovum forceps
Correct Ans: A
Explanation

Ans. A. Episiotomy scissor

  • It is Braun episiotomy scissor.
  • It is non-ratcheted, finger ring scissors, angled with smooth, blunt/blunt tips, and has a length of 5-1/2 inches.

Question.32
Identify the given procedure done in labor room:

AIIMS PG May-2018
A. NST
B. Amniotic fluid index
C. Amniocentesis
D. MCA Doppler
Correct Ans: A
Explanation

Ans. A. NST
[Ref: William’s Obstetrics, 24’h ed]

  • Answer should be CTG as the picture shown has tocoprobe but since that’s not in the option then the next best answer will be NST since both measure the same parameters.
  • nonstress test (a procedure that measures the fetal heart rate in response to fetal movements)

Question.33
Identify the anamoly based on the image below

AIIMS PG May-2018
A. Twin-twin transfusion syndrome
B. Monochorionic diamniotic pregnancy
C. Monochorionic monoamniotic pregnancy
D. Dichorionic diamniotic pregnancy
Correct Ans: C
Explanation

Ans: C. Monochorionic monoamniotic pregnancy
[Ref: Williams’s Obstetrics, 24’h ed]
Two umbilical cords are seen arising from the same placenta and there is no intervening amniotic membrane, it is monochorionic monoamniotic placenta.


Question.34

A child underwent Kasai’s portoenterostomy for management of EHBA; apart from the relations depicted in the given image which one of the following is not associated with this condition?

AIIMS PG May-2018
A. Renal agenesis
B. Malrotation of gt
C. Polysplenia
D. Aberrant right hepatic artery
Correct Ans: A
Explanation

Ans: A. Renal agenesis

  • EHBA/ Extra Hepatic Biliary Atresia is characterized by obliteration or discontinuity of the extrahepatic biliary system, resulting in obstruction to bile flow.
  • Roux-en-Y hepatic portoenterostomy procedure (Kasai Procedure) – Standard initial operation for treatment of infants with biliary atresia

Question.35
A 4-year-old child presented with painless genu yarum. X-ray  bilateral knee was done and is shown below. Which of the fotlowing is most likely diagnosis?

AIIMS PG May-2018
A. Rickets
B. Scurvy
C. Congenital anomaly
D. Trauma to epiphysis causing degeneration
Correct Ans: A
Explanation

Ans: A. Rickets
[Ref Essential Orthopaedics, 5’h ed”; pg. 311]
The history of painless genu varum along with the given X-ray findings are suggestive of Rickets.


Question.36

A 3-year-old child presented with umbilical cord discharge as shown in the image, with no palpable mass in the abdomen. What is the best step?

AIIMS PG May-2018
A. USG Abdomen
B. Diagnostic laparotomy and proceed
C. MRI abdomen
D. CT
Correct Ans: B
Explanation

Ans: B. Diagnostic laparotomy and proceed
[Ref: Corun Pediatric Surgery, Vh ed., pg. 961]

  • The given image and history is suggestive of umbilical polyp.

Umbilical polyp:

  • Fine masses comprised of intestinal epithelium or uroepithel ium, which are omphalomesentenc remnants. They usually present with discharge.
  • Umbilical polyps are rare and often larger than granulomas, do not respond to silver nitrate therapy
  • Surgical exploration is the definitive diagnostic test.

Question.37
what does “A” depict in the given image of serum immunoglobulin Levels in a paediatric patient? 

AIIMS PG May-2018
A. Age(years)
B. IgG
C. Maternal and Fetal IgM
D. Maternal and Fetal IgG
Correct Ans: D
Explanation

Ans: D. Maternal and Fetal IgG


Question.38

What is the treatment modelity for the following fracture?

AIIMS PG May-2018
A. Tension band wiring
B. Plating
C. External fixation
D. ORIF
Correct Ans: A
Explanation

Ans: A. Tension band wiring
Tension band principle

  • Distractive forces are converted into compressive forces to heal fracture site. Here distractive forces caused by quadriceps pull convert into compression at fracture site.
  • There are two types of Tension band wiring
  • Dynamic e.g. Fracture at Patella, Olecranon etc
  • Static e.g. Fracture at medial malleolous, GT of femur

Question.39
Which fracture results in the given deformity?

AIIMS PG May-2018
A. Supracondylar fracture of humerus
B. Lateral condylar fracture
C. Olecranon fracture
D. Radial head fracture
Correct Ans: A
Explanation

Ans: A. Supracondylar fracture of humerus
[Ref: Apley’s system of orthopaedics and fracture 9′ ed. pg. 750,758-760]

  • The deformity shown in above picture is cubitus varus deformity.
  • Most common cause of cubitus varus deformity is malunited supracondylar fracture of humerus .

Question.40
Characteristic of venous blood flow?

AIIMS PG May-2018
A. Monophasic
B. Biphasic
C. Triphasic
D. Non phasic
Correct Ans: A
Explanation

Ans: A. Monophasic

Triphasic: forward flow in systole reverse flow in late systole / early diastole forward flow in late diastole
Biphasic: having two phases or variations having forward and reverse flow 7 reverse flow in diastole steady positive flow in the diastole, or forward flow in systole
Monophasic: single phase with slow (dampened flow) acceleration/deceleration low velocity high velocity

%d bloggers like this:
Malcare WordPress Security